Química para ingeniería: problemas y prácticas [2 ed.]
 9788490481097

Citation preview

María José Climent Olmedo Belén Ferrer Ribera María Luisa Marín García Sergio Navalón Oltra

Susana Encinas Perea Sara Iborra Chornet Isabel Morera Bertomeu Félix Sancenón Galarza

 

     2ª edición

   



Primera edición, 2012 Segunda edición, 2013

© María José Climent Olmedo Susana Encinas Perea Belén Ferrer Ribera Sara Iborra Chornet María Luisa Marín García Isabel Morera Bertomeu Sergio Navalón Oltra Félix Sancenón Galarza © de la presente edición: Editorial Universitat Politècnica de València  Tel. 96 387 70 12 / www.lalibreria.upv.es / Ref.: 949 Imprime: Byprint Percom, sl ���������������������������

ISBN: 978-84-9048-109-7 Impreso bajo demanda Queda prohibida la reproducción, distribución, comercialización, transformación, y en general, cualquier otra forma de explotación, por cualquier procedimiento, de todo o parte de los contenidos de esta obra sin autorización expresa y por escrito de sus autores. Impreso en España

Prólogo El libro “Química para Ingeniería. Problemas y Prácticas” complementa al texto teórico que lleva por título “Química para Ingeniería”. Contiene problemas y prácticas de laboratorio de Química General y Orgánica, y está dedicado a los estudiantes que cursan Química en el primer curso de las diversas titulaciones de Grado de Ingeniería. Con este texto se pretende que los alumnos completen su aprendizaje básico de la Química mediante la aplicación de los conocimientos teóricos a diversos problemas prácticos y experimentales. Consta de una primera parte compuesta de problemas, agrupados en capítulos, que se correlacionan con los temas del libro de teoría “Química para Ingeniería”. Incluye problemas de termodinámica y equilibrio químico, de cinética y de reacciones de oxidación y reducción. Asimismo se presentan problemas relacionados con la Química Orgánica organizados según las familias orgánicas. En los diferentes capítulos se incluyen una serie de problemas resueltos, seleccionados de menor a mayor dificultad, para aplicar todos los conceptos químicos estudiados en las clases teóricas, indicando de forma detallada y razonada el procedimiento a seguir para su resolución. A continuación, se presentan una serie de problemas representativos de cada tema, que se resolverán en el aula. También se incluyen varias colecciones de problemas propuestos, agrupando diversos temas, para que los alumnos los resuelvan en pequeños grupos, fomentando así el aprendizaje cooperativo con el fin de que el estudiante adquiera las habilidades de trabajo en equipo, que le serán de gran utilidad en su futuro ejercicio profesional. Para estos problemas se aporta el resultado numérico, con el fin de estimular al alumno a su resolución. La segunda parte de este texto se compone de seis prácticas para realizar en el laboratorio de Química, que se adecuan a los contenidos teóricos impartidos. Las dos primeras prácticas tratan la termodinámica y cinética química, respectivamente. La práctica siguiente aborda las aplicaciones industriales de las reacciones de oxidación y reducción. A continuación, se trata el estudio de la relación entre la estructura molecular y las propiedades físicas de los compuestos orgánicos e inorgánicos. Y para finalizar, las dos últimas prácticas están dedicadas a la síntesis de ésteres con propiedades aromatizantes y a la preparación de distintos polímeros como la Baquelita, el Nylon 6,6, etc. Si bien el libro ha sido elaborado cuidadosamente, cualquier comentario que permita mejorarlo será bien recibido para una posterior revisión. Julio de 2013 Los autores 3

Índice Parte I. Problemas Capítulo 1. Termodinámica y Equilibrio químico ........................................... 9 Problemas resueltos ...................................................................... 11 Problemas para resolver ................................................................ 38 Capítulo 2. Cinética química .......................................................................... 43 Problemas resueltos ...................................................................... 45 Problemas para resolver ................................................................ 55 Capítulo 3. Problemas propuestos de Termodinámica, Equilibrio químico y Cinética química ......................................................... 59 Capítulo 4. Reacciones de Oxidación y Reducción . .................................... 75 Problemas resueltos ...................................................................... 77 Problemas para resolver ................................................................ 93 Capítulo 5. Problemas propuestos de Reacciones de Oxidación y Reducción ................................................................................. 99 Capítulo 6. Introducción a la Química Orgánica ........................................ 111 Problemas resueltos .................................................................... 113 Problemas para resolver .............................................................. 117 Capítulo 7. Hidrocarburos Saturados e Insaturados ................................. 119 Problemas resueltos .................................................................... 121 Problemas para resolver .............................................................. 128 Capítulo 8. Hidrocarburos Aromáticos ....................................................... 133 Problemas resueltos .................................................................... 135 Problemas para resolver .............................................................. 147 Capítulo 9. Problemas propuestos de Hidrocarburos Saturados, Insaturados y Aromáticos. ........................................................ 151

55

Química para Ingeniería. Problemas y prácticas

Capítulo 10. Alcoholes, Éteres y Epóxidos. ................................................ 159 Problemas resueltos .................................................................. 161 Problemas para resolver ............................................................ 167 Capítulo 11. Aldehídos y Cetonas ............................................................... 169 Problemas resueltos .................................................................. 171 Problemas para resolver ............................................................ 175 Capítulo 12. Ácidos carboxílicos y Derivados. ........................................... 177 Problemas resueltos .................................................................. 179 Problemas para resolver ............................................................ 184 Capítulo 13. Problemas propuestos de Alcoholes, Éteres, Epóxidos, Aldehídos, Cetonas, Ácidos y Derivados. ........... 187

Parte II. Prácticas Práctica 1. Termodinámica Química ............................................................... 195 Práctica 2. Cinética Química ........................................................................... 203 Práctica 3. Aplicaciones Industriales de las Reacciones de Oxidación y reducción............................ ........................................................ 209 Práctica 4. Relación entre Estructura Molecular y Propiedades Físicas de los Compuestos Químicos ........................................................ 223 Práctica 5. Preparación de Polímeros ............................................................. 235 Práctica 6. Preparación de Ésteres. Síntesis del Acetato de Isoamilo ............ 247

66



Parte I. Problemas

1



                

3



Problemas resueltos 1. Calcular la Eo a 298 K para la siguiente reacción: N2 (g) + 2 O2 (g) � N2O4 (g) sabiendo que a 298 K y 1 atmósfera de presión, la Ho = 9,16 kJ. Resolución: Para calcular la variación de energía interna en condiciones estándar (Eo) utilizamos la expresión que la relaciona con la variación de entalpía (Ho): Ho = Eo + (P·V) La variación de presión y volumen ((P·V)) en una reacción en la que sólo intervienen gases que se comportan de una manera ideal se puede expresar como: (P·V) = n(R·T) así, se puede escribir: Ho = Eo + n(R·T)

[I]

donde: n: variación algebraica de los coeficientes estequiométricos de los gases R: constante de los gases ideales (8,31 J mol-1 K-1) T: temperatura (Kelvin) Sustituyendo los valores en [I] y despejando Eo se tiene: 9160 = Eo + (1-3) (8,31·298) Eo = 14112,8 J mol-1 = 14,1 kJ mol-1

2. En la tabla se muestran los calores específicos de varios metales:

4



Al

Fe

Cu

 ( J oC-1 g-1)

0,903

0,449

0,385

11

Química para Ingeniería. Problemas y prácticas



Si se quiere construir un depósito para calentar rápidamente una determinada cantidad de agua, ¿cuál de los tres metales será más adecuado? Resolución: El calor específico de una sustancia es la cantidad de calor necesaria para aumentar un grado la temperatura de un gramo de dicha sustancia. Así, el recipiente que transferirá más calor, y calentará más rápidamente el agua, será el del metal con mayor calor específico, es decir, el aluminio.

3. a) ¿Qué cantidad de calor se necesitará para aumentar 60 oC la temperatura de 1,5 l de agua que inicialmente se encuentra a 20 oC? b) ¿Cuántos gramos de butano habrá que quemar para realizar dicha transformación? Datos: Hc (C4H10, g) = - 11 kcal g-1; Cp (H2O, l) = 18,1 cal grado-1 mol-1 Mr (H2O) = 18; d (H2O) = 1 g ml-1 Resolución: a) El proceso que tiene lugar se puede representar como: H2O (l, 20 oC) � H2O (l, 80 oC) Se trata de un proceso físico a presión constante en el que habrá que suministrar energía para calentar el agua desde 20 oC hasta 80 oC. Para calcular la cantidad de calor necesaria habrá que tener en cuenta que la capacidad calorífica a presión constante (Cp) se puede expresar como: Cp 

dqp dT

y que la variación de entalpía es el calor a presión constante (H = qp). Despejando, se tiene que: H = Cp·T

12

5

Capítulo 1. Termodinámica y Equilibrio químico



para “n” moles: H = n·Cp·T = n·Cp·(T2 – T1)

[II]

donde: n: número de moles de agua que se calientan (n = masa (g) / Mr) Cp: capacidad calorífica a presión constante del agua líquida T1: temperatura inicial del agua T2: temperatura final del agua Sustituyendo en la ecuación [II] se obtiene la cantidad de calor necesaria para calentar el agua: H = (1500 / 18)·18,1·(353 – 293) = 90500 cal H = 90,5 kcal b) Teniendo en cuenta que la entalpía de combustión del butano (Hc) es -11 kcal g-1, para obtener las 90,5 kcal necesarias para calentar el agua del apartado anterior, la cantidad de butano a quemar es: masa de butano = 90,5 · (1 g de butano/11 kcal) m = 8,23 g de butano

4. El metano se puede obtener a partir del gas de síntesis según la siguiente reacción: CO (g) + 3 H2 (g) � CH4 (g) + H2O (l) a) Calcular la variación de entalpía molar (Ho) a 298 K para la reacción de obtención del metano. b) Determinar la variación de entalpía del proceso cuando se obtienen 20 g de metano. c) Calcular Ho a 700 oC para la reacción del apartado (b). d) Determinar la variación de entalpía, a 700 oC, si la reacción de obtención de 20 g de metano se produce en un 60%. Datos a 298 K: Hof (kJ mol-1): CO (g) = - 110,5; CH4 (g) = - 74,8; H2O (l) = - 285,8 Cp (cal grado-1 mol-1): CO (g) = 7,0; H2 (g) = 6,9; CH4 (g) = 9,0; H2O (l) = 18,1 6

13

Química para Ingeniería. Problemas y prácticas



Ar (C) = 12; Ar (H) = 1 Resolución: a) Según la reacción estequiométrica y a partir de los datos de Hof se puede obtener el valor de la entalpía de la reacción (Ho) a 298 K, aplicando la expresión: Ho =  n·Hof (productos) –  n·Hof (reactivos) donde: Ho: calor absorbido o desprendido en la reacción a presión constante Hof (productos): entalpías de formación de productos Hof (reactivos): entalpías de formación de reactivos n: coeficiente estequiométrico de cada una de las especies de la reacción ajustada Así, multiplicando el valor de la entalpía de formación de cada especie por su correspondiente coeficiente estequiométrico, se tiene: Ho = Hof (CH4, g) + Hof (H2O, l) – Hof (CO, g) – 3·Hof (H2, g) sustituyendo los valores y teniendo en cuenta que Hof (H2, g) = 0, se obtiene: Ho = - 74,8 - 285,5 + 110,5 Ho = - 250,1 kJ mol-1 b) Teniendo en cuenta que la masa molecular del metano es 16, el número de moles de este gas que corresponden a 20 g es: n = 20/16 = 1,25 moles de metano por lo tanto, la entalpía de la reacción cuando se obtienen 20 g de metano es: Ho = (- 250,1 kJ mol-1)·1,25 mol Ho = - 312,6 kJ c) Para calcular la entalpía de la reacción a 700 oC (973 K) es necesario aplicar la ley de Kirchoff a la reacción: CO (g) + 3 H2 (g) � CH4 (g) + H2O (l) así:

Ho (973 K )  Ho (298 K )  14



973 298

(n  Cp )  dT

[III]

7

Capítulo 1. Termodinámica y Equilibrio químico



donde: Ho (973): entalpía de la reacción a 973 K Ho (298): entalpía de la reacción a 298 K (n·Cp) =  n·Cp (productos) –  n·Cp (reactivos), siendo n el coeficiente estequiométrico de cada una de las especies en la reacción ajustada y Cp la capacidad calorífica a presión constante de cada uno de los productos y reactivos que intervienen T: temperatura Sustituyendo los valores, unificando las unidades en calorías, en [III] se tiene: Ho (973  )  250,1  1000

1  4,18



973 298

(9,0  18,1  7,0  3  6,9)  dT

Ho (973  )  59832,5  0,6 



973 298

dT

Ho (973  )  59832,5  0,6  (973  298) Ho (973 K) = - 60237.5 cal Ho (973 K) = - 251,8 kJ Teniendo en cuenta que se obtienen 1,25 moles de metano, la entalpía de la reacción es: Ho (973 K) = 1,25 · (- 251,8) Ho (973 K) = - 314,7 kJ d) Si el rendimiento de la reacción es del 60%, el calor desprendido a 973 K es: Ho (973 K) = 0,6 · (- 314,7) Ho (973 K) = - 188,8 kJ

5. Un recipiente metálico contiene 49 l de acetileno a una atmósfera de presión y a 25 oC. Si se produce su combustión en condiciones adiabáticas, en presencia de la cantidad estequiométrica de aire y a presión constante, ¿se podrá fundir un trozo de hierro?

8

15

Química para Ingeniería. Problemas y prácticas



Datos a 25 oC: Hof (kcal mol-1): acetileno (g) = 54,4; agua (v) = - 57,8; dióxido de carbono (g) = - 94,1 Cp (cal mol-1 grado-1): acetileno (g) = 10,5; dióxido de carbono (g) = 8,9 agua (v) = 8,1; oxígeno (g) = 7,1; nitrógeno (g) = 7,0 Considerar que el aire contiene 20% de oxígeno y 80% de nitrógeno. Tfusión hierro = 1600 oC Resolución: La reacción ajustada que tiene lugar es la siguiente: C2H2 (g) + 5/2 O2 (g) � 2 CO2 (g) + H2O (v) Para saber si se puede fundir el hierro es necesario calcular la temperatura máxima de llama para un proceso a presión constante y adiabático. Si ésta es superior a la temperatura de fusión del hierro, sí que podemos fundirlo. Para calcular la temperatura máxima de llama se considera que el calor liberado en la reacción de combustión va a ser absorbido por las especies finales (tanto los productos formados como los reactivos en exceso), elevando la temperatura inicial hasta la temperatura final o “máxima de llama”. Así, el calor desprendido en la reacción de combustión más el absorbido por las especies finales será igual a cero, es decir: Hreacción + Habsorbido = 0

 Hreacción  Habsorbido 



T max Ti

n  Cp  dT

[IV]

donde: Hreacción: calor desprendido en la reacción a presión constante n: número de moles de los productos remanentes (productos formados y reactivos en exceso) Cp: capacidad calorífica a presión constante de los productos remanentes Ti: temperatura inicial Tmax: temperatura final o temperatura máxima de llama Así pues, el primer paso va a ser el cálculo del calor que se desprende en la reacción de combustión del acetileno. Para calcular la entalpía molar de la 16

9

Capítulo 1. Termodinámica y Equilibrio químico



reacción de combustión del acetileno partiendo de las entalpías de formación a 25 oC se utiliza la expresión: Hoc =  n·Hof (productos) –  n·Hof (reactivos) donde: Ho: calor desprendido en la reacción de combustión a presión constante Hof (productos): entalpías de formación de productos Hof (reactivos): entalpías de formación de reactivos n: coeficiente estequiométrico de cada una de las especies de la reacción ajustada Multiplicando el valor de la entalpía de formación de cada especie por su correspondiente coeficiente estequiométrico, se tiene: Hoc = Hof (H2O, v) + 2·Hof (CO2, g) – Hof (C2H2, g) – 5/2·Hof (O2, g) sustituyendo los valores se obtiene: Ho = (- 57,8) + 2·(- 94,1) – (54,4) Ho (298 K) = - 300,4 kcal mol-1 El número de moles de acetileno que tenemos en el recipiente se calcula a partir de la ecuación del gas ideal, según: P·V = n·R·T donde: P: presión ejercida por el gas (atmósferas) V: volumen ocupado por el gas (litros) n: número de moles del gas R: constante de los gases ideales T: temperatura (Kelvin) Sustituyendo los valores, tenemos que: 1·49 = n·0,082·298 n = 2 moles de acetileno Teniendo en cuenta que el calor desprendido cuando reacciona completamente un mol de acetileno es -300,4 kcal; cuando reaccionan dos moles de dicho gas

10

17

Química para Ingeniería. Problemas y prácticas



el calor desprendido es: Hreacción = 2 · (- 300,4) = - 600,8 kcal Para determinar la temperatura máxima de llama se necesita conocer el valor de n·Cp de los productos después de la combustión (ecuación [IV]) y para ello se plantea el siguiente balance molar: C2H2

+

5/2 O2



2 CO2

+

H2O

Moles iniciales

2

5

-

-

Moles reaccionados

2

5

-

-

Moles finales

-

-

4

2

Como el oxígeno se introduce en forma de aire, cuya composición es 20% de oxígeno y 80% de nitrógeno, también se está introduciendo nitrógeno que va a absorber parte del calor generado en la reacción. Así se tiene: moles de nitrógeno = 4·(moles de oxígeno) = 4·5 = 20 Del balance molar, y teniendo en cuenta el nitrógeno que se introduce en la reacción, se deduce que:  n·Cp = 4·Cp (CO2) + 2·Cp (H2O) + 20·Cp (N2) = 191,8 cal grado-1 Sustituyendo en la ecuación [IV] se tiene:

600,8  1000 



Tmax 298

191,8  dT  191,8 



Tmax 298

dT  191,8  (Tmax  298)

de donde: Tmax = 3430,4 K = 3157,4 oC Así pues, como Tmax (3157,4 oC) > Tfusión hierro (1600 oC) sí que se podrá fundir el hierro.

6. A 298 K se introducen en un recipiente no dilatable de 100 l de capacidad, cierta cantidad de gas propano y el oxígeno estequiométricamente necesario para que la combustión sea completa, siendo la presión total de la mezcla inicial de 25,42 atmósferas. Una chispa provoca la explosión.

18

11

Capítulo 1. Termodinámica y Equilibrio químico



a) Calcular la temperatura final del sistema. b) Calcular la presión máxima alcanzada en el recipiente tras el proceso. Nota: Considérese que el agua se obtiene en estado vapor. Datos a 298 K: Hof (kcal mol-1): CO2 (g) = - 94,0; H2O (v) = - 57,8; propano (g) = - 104,7 Cp (cal grado-1 mol-1): CO2 (g) = 8,9; N2 (g) = 7,0; H2O (v) = 8,1 Resolución: a) La reacción ajustada que tiene lugar es la siguiente: C3H8 (g) + 5 O2 (g) � 3 CO2 (g) + 4 H2O (g) Se debe calcular la temperatura máxima de explosión para un proceso a volumen constante y adiabático. Para calcular la temperatura máxima de explosión tenemos que considerar que el calor liberado en la reacción de combustión va a ser absorbido por las especies finales (tanto los productos formados como los reactivos en exceso), elevando la temperatura inicial hasta la temperatura final o “máxima de explosión”. Así, el calor desprendido en la reacción de combustión más el absorbido por las especies finales será igual a cero, es decir: Ereacción + Eabsorbido = 0

 Ereacción  Eabsorbido 



T max Ti

n  Cv  dT

[V]

donde: Ereacción: calor desprendido en la reacción a volumen constante n: número de moles de los productos remanentes (productos formados y reactivos en exceso) Cv: capacidad calorífica a volumen constante de los productos remanentes Ti: temperatura inicial Tmax: temperatura final o temperatura máxima de explosión Se trata de un proceso a volumen constante y el siguiente paso es calcular Eo. Para ello se necesita conocer primero Ho de la reacción de combustión del propano que se calcula a partir de las entalpías de formación a 25 oC: Ho =  n·Hof (productos) –  n·Hof (reactivos)

12

19

Química para Ingeniería. Problemas y prácticas



donde: Ho: calor desprendido en la reacción de combustión a presión constante Hof (productos): entalpías de formación de productos Hof (reactivos): entalpías de formación de reactivos n: coeficiente estequiométrico de cada una de las especies en la reacción ajustada. Así, multiplicando el valor de la entalpía de formación de cada especie por su correspondiente coeficiente estequiométrico, se tiene: Hoc = 4·Hof (H2O, v) + 3·Hof (CO2, g) – Hof (C3H8, g) – 5·Hof (O2, g) sustituyendo los valores se obtiene: Ho = 4·(- 57,8) + 3·(- 94,1) – (- 104,7) Ho (298 K) = - 408,5 kcal mol-1 Para calcular la variación de energía interna de la reacción de combustión del propano se emplea la ecuación [I]: Ho = Eo + n(R·T) donde: n: variación de los coeficientes estequiométricos de los gases (n = nproductos – nreactivos) R: constante de los gases ideales (1,98 cal mol-1 K-1) T: temperatura (Kelvin) Sustituyendo los valores en [I] y despejando Eo se tiene: - 408500 = Eo + (7- 6) (1,98 · 298) Eo = - 409100 cal mol-1 = - 409,1 kcal mol-1 A continuación se calcula el número de moles totales que se introducen en el reactor, considerando que la mezcla se comporta como un gas ideal: P·V = n·R·T donde: P: presión ejercida por el gas (atmósferas) V: volumen ocupado por el gas (litros) n: número de moles del gas

20

13

Capítulo 1. Termodinámica y Equilibrio químico



R: constante de los gases ideales T: temperatura (Kelvin) Sustituyendo los valores tenemos que: 25,42·100 = n·0,082·298 n = 104,03 moles totales Se puede calcular los moles de propano y de oxígeno sabiendo que la relación estequiométrica entre ambos gases es 1:5: moles propano = 104,03/6 = 17,34 moles oxígeno = 104,03 – 17,34 = 86,96 Teniendo en cuenta que el calor desprendido cuando reacciona completamente un mol de propano es - 409,1 kcal, cuando reaccionan 17,34 moles de dicho gas el calor desprendido es: Ereacción = 17,34 · (- 409,1) = - 7093,8 kcal Para determinar la temperatura máxima de llama se necesita conocer el valor de n·Cv de los productos después de la reacción de combustión del propano (ecuación [V]) y para ello, se plantea el siguiente balance molar: C3H8

+

5 O2



3 CO2

+

4 H2O

Moles iniciales

17,34

86,92

-

-

Moles reaccionados

17,34

86,92

-

-

Moles finales

-

-

52,02

69,36

Los valores de Cv de las especies remanentes después de la combustión se calculan a partir de la relación de Meyer: C p = Cv + R así, considerando R = 1,98 cal mol-1 grado-1 se tiene: Cv (CO2) = 6,9 cal grado-1 mol-1; Cv (H2O) = 6,1cal grado-1 mol-1 del balance molar se deduce que:  n·Cv = 52,02 · Cv (CO2) + 69,36 · Cv (H2O) = 782,03 cal grado-1

14

21

Química para Ingeniería. Problemas y prácticas



Sustituyendo en la ecuación [V] tenemos:

7093,8  1000 



T max 298

782,03  dT  782,03 



T max 298

dT  782,03  (Tmax  298)

de donde: Tmax = 9370 K b) Para calcular la presión máxima de explosión utilizamos la ecuación de los gases ideales para las condiciones finales de la reacción: Pmax·V = nT·R·Tmax donde: Pmax: presión total que ejercen los gases que quedan al final de la reacción o presión máxima de explosión V: volumen ocupado por el gas (litros) nT: número total de moles de los gases después de la reacción R: constante de los gases ideales Tmax: temperatura máxima de explosión Sustituyendo los valores, y teniendo en cuenta que: nT = nCO2 + nH2O = 52,02 + 69,36 = 121,38 moles y Tmax = 9370 K, se obtiene: Pmax = (121,38·0,082·9370)/100 Pmax = 932,6 atm

7. Cuando se hace reaccionar metano y vapor de agua a 800 oC, se obtiene una mezcla de hidrógeno y monóxido de carbono que se denomina gas de síntesis o gas de agua. a) Determinar Kp para el proceso a 800 oC. b) Si en un reactor, a 800 oC, se introducen 3 moles de metano, 3 de vapor de agua, 2 de monóxido de carbono y 1 de hidrógeno, siendo la presión total de la mezcla 10 atmósferas, ¿en qué sentido tendrá lugar la reacción? Justificar la respuesta. c) Si una vez alcanzado el equilibrio se aumenta la temperatura del sistema, explicar cómo se ve afectado el rendimiento de obtención del gas de síntesis. 22

15



Capítulo 1. Termodinámica y Equilibrio químico

d) ¿En qué sentido se desplazará el equilibrio si aumentamos la presión total del sistema? Justificar la respuesta. Datos a 25 oC: Hof (kcal mol-1): CH4 (g) = - 17,9; H2O (v) = - 57,8; CO (g) = - 26,1 So (cal grado-1 mol-1): CH4 (g) = 44,7; H2O (v) = 45,3; CO (g) = 47,3; H2 (g) = 28,5 Considerar Cp = 0 Resolución: a) La reacción de formación del gas de síntesis o del gas de agua es: CH4 (g) + H2O (g)  3 H2 (g) + CO (g) o

La Kp a 800 C (1073 K) se puede calcular a partir del valor de Go a dicha temperatura mediante la expresión: Go (1073 K) = – R·T·lnKp

[VI]

donde: Go: energía libre de Gibbs R: constante de los gases ideales (cal mol-1 grado-1 o J mol-1 grado-1) T: temperatura (Kelvin) Kp: constante de equilibrio Para calcular Go a 1073 K se debe emplear la siguiente expresión: Go (1073 K) = Ho (1073 K) – T·So (1073 K)

[VII]

Los datos de que se dispone están tabulados a 298 K y se puede calcular a partir de ellos Ho (298 K) y So (298 K). En primer lugar se calcula Ho (298 K) de la reacción empleando la expresión: Ho (298 K) =  n·Hof (productos) –  n·Hof (reactivos) donde: Ho: calor absorbido o desprendido en la reacción a presión constante Hof (productos): entalpías de formación de productos Hof (reactivos): entalpías de formación de reactivos n: coeficiente estequiométrico de cada una de las especies de la reacción ajustada. 16

23

Química para Ingeniería. Problemas y prácticas



Así, multiplicando el valor de la entalpía de formación de cada especie por su correspondiente coeficiente estequiométrico, se tiene: Ho (298 K) = Hof (CO, g) + 3·Hof (H2, g) – Hof (CH4, g) – Hof (H2O, v) sustituyendo los valores y teniendo en cuenta que Hof (H2, g) = 0, se obtiene: Ho (298 K) = (-26,1) – (-17,9) – (-57,8) Ho (298 K) = 49,6 kcal mol-1 Puesto que se indica que Cp = 0, la entalpía de la reacción es constante con la temperatura y así: Ho (1073 K) = 49,6 kcal mol-1 En segundo lugar, para determinar So (298 K) de la reacción utilizaremos la expresión: So (298 K) =  n·So (productos) –  n·So (reactivos) donde: So: incremento de entropía en la reacción a presión constante So (productos): entropías absolutas de los productos So (reactivos): entropías absolutas de los reactivos n: coeficiente estequiométrico de cada una de las especies de la reacción ajustada De esta forma se puede calcular So (298 K) teniendo en cuenta los valores de las entropías absolutas de las especies que participan en la reacción: So (298 K) = So (CO, g) + 3·So (H2, g) – So (CH4, g) – So (H2O, v) sustituyendo los valores se obtiene: So (298 K) = (47,3 + 3·28,5) – (44,7 + 45,3) So (298 K) = 42,8 cal mol-1 grado-1 puesto que Cp = 0, se considera que: So (1073 K) = 42,8 cal mol-1 grado-1 A partir de los valores calculados de Ho (1073 K) y de So (1073 K) calculamos Go (1073 K) aplicando la ecuación [VII]: Go (1073 K) = Ho (1073 K) – T·So (1073 K)

24

17

Capítulo 1. Termodinámica y Equilibrio químico



Go (1073 K) = 49600 – 1073·42,8 Go (1073 K) = 3675,6 cal mol-1 Por último, para calcular la constante de equilibrio (Kp) de la reacción empleamos la ecuación [VI]: Go (1073 K) = – R·T·lnKp 3675,6 = – 1,98·1073·lnKp Kp = 0,18 b) Para saber si la reacción va a ser espontánea en las condiciones del enunciado se calcula el valor de G a 1073 K mediante la expresión:

G (1073 K )  Go (1073 K )  R  T  ln

PCO  PH32

PH2 O  PCH4

[VIII]

Para ello se necesita calcular los valores de las presiones parciales de reactivos y productos empleando la ley de Dalton: Pi = xi·PT siendo: Pi: presión parcial de cada uno de los gases que intervienen en la reacción xi: fracción molar de cada uno de los gases que intervienen en la reacción calculada a partir de los moles de cada gas y los moles totales (xi = ni / nT) PT: presión total de la mezcla de gases Sabiendo que nT = nCH4 + nH2O + nCO + nH2 = 3 + 3 + 2 + 1 = 9 De esta manera se puede escribir:

PCH4  xCH4  PT 

3  10  3,3 atm 9

PH2 O  xH2 O  PT 

3  10  3,3 atm 9

PCO  xCO  PT 

18

2  10  2,2 atm 9

25

Química para Ingeniería. Problemas y prácticas

PH2  xH2  PT 



1  10  1,1 atm 9

Sustituyendo los valores de las presiones parciales en la ecuación [VIII] se obtiene la variación de la energía libre de Gibbs en las condiciones indicadas para la reacción: G (1073 K) = 3675,6 + 1,98·1073·ln [(2,2·1,13) / (3,3·3,3)] G (1073 K) = 885,1 cal mol-1 Como G (1073 K) > 0 la reacción no es espontánea tal como está escrita y transcurre de derecha a izquierda. c) Como en la reacción de obtención del gas de síntesis se tiene que Ho (1073 K) = 49,6 kcal mol-1, la reacción es endotérmica, por tanto, un aumento de temperatura desplazará el equilibrio hacia la derecha y favorecerá la obtención del gas de síntesis. d) Si aumentamos la presión, el equilibrio se desplaza hacia donde hay menos número de moles, en este caso hacia los reactivos. Por lo tanto el equilibrio se desplazará de derecha a izquierda.

8. En la siguiente reacción: A (g)  B (g) + 2 C(g) se sabe que ha reaccionado el 30% del compuesto A cuando la temperatura es de 200 oC y el 50% cuando es 250 oC. Sabiendo que la presión total en el equilibrio es 1 atmósfera. a) Determinar Kp para el proceso a 200 oC y a 250 oC. b) Calcular la temperatura para la cual ha reaccionado el 70% del compuesto A y los valores de Go, Ho e So a esa temperatura. Nota: Considerar que Cp = 0

26

19

Capítulo 1. Termodinámica y Equilibrio químico



Resolución: a) Para calcular la Kp de la reacción a 200 oC (473 K) se utiliza la expresión de la constante de equilibrio en función de las presiones parciales, según: K p ( 473 K ) 

PC2  PB PA

[IX]

Según la ley de Dalton: PC = xC·PT PB = xB·PT PA = xA ·PT Para calcular las fracciones molares de los tres componentes de la reacción llamamos “no” al número de moles iniciales de A (g), y teniendo en cuenta que en el equilibrio ha reaccionado el 30% del mismo, se plantea el siguiente balance molar teniendo en cuenta la estequiometría de la reacción: A (g)



B (g)

+

2 C (g)

Moles iniciales

no

-

-

Moles reaccionados

0,3no

-

-

Moles equilibrio

0,7no

0,3no

0,6no

El número de moles totales en el equilibrio (nT) es: nT = 0,7no + 0,3no + 0,6no = 1,6no Teniendo en cuenta la ley de Dalton y que la presión total en el equilibrio es de 1 atmósfera se puede escribir:

20

PA  x A  PT 

0,7n o  1  0,44 atm 1,6n o

PB  x B  PT 

0,3n o  1  0,19 atm 1,6n o

PC  x C  PT 

0,6n o  1  0,40 atm 1,6n o

27

Química para Ingeniería. Problemas y prácticas



Sustituyendo los valores de las presiones parciales en la expresión de la constante Kp, tendríamos: K p ( 473 K ) 

PC2  PB (0,4 ) 2  0,19  PA 0,44

Kp (473 K) = 0,07 Para calcular Kp a 250 oC (523 K) empleamos el mismo procedimiento pero teniendo en cuenta que en el equilibrio ha reaccionado el 50% de A (g) y que la presión total es de 1 atmósfera. Así, el balance molar será: A (g)



B (g)

+

2 C (g)

Moles iniciales

no

-

-

Moles reaccionados

0,5no

-

-

Moles equilibrio

0,5no

0,5no

1,0no

El número de moles totales en el equilibrio (nT) es, en este caso: nT = 0,5no + 0,5no + 1,0no = 2,0no Teniendo en cuenta la ley de Dalton: PA  x A  PT 

0,5n o  1  0,25 atm 2n o

PB  x B  PT 

0,5no  1  0,25 atm 2no

PC  x C  PT 

1n o  1  0,50 atm 2n o

Así, sustituyendo los valores de las presiones parciales en la expresión [IX] calculamos el valor de Kp (523 K): K p ( 523 K ) 

PC2  PB (0,5 )2  0,25  PA 0,25

Kp (523 K) = 0,25

28

21

Capítulo 1. Termodinámica y Equilibrio químico



b) Para calcular la temperatura a la cual ha reaccionado el 70% de A (g) debemos obtener el valor de la constante de equilibrio (Kp) teniendo en cuenta el correspondiente balance molar, y que la presión total en el equilibrio es 1 atmósfera: A (g)



B (g)

+

2 C (g)

Moles iniciales

no

-

-

Moles reaccionados

0,7no

-

-

Moles equilibrio

0,3no

0,7no

1,4no

El número de moles totales en el equilibrio (nT) es, en este caso: nT = 0,3no + 0,7no + 1,4no = 2,4no Teniendo en cuenta la ley de Dalton y que la presión total en el equilibrio es de 1 atmósfera podemos escribir: PA  x A  PT 

0,3n o  1  0,125 atm 2,4n o

PB  x B  PT 

0,7n o  1  0,292 atm 2,4n o

PC  x C  PT 

1,4n o  1  0,583 atm 2,4n o

De nuevo, sustituyendo los valores de las presiones parciales en la ecuación [IX] calculamos el valor de Kp: Kp 

PC2  PB ( 0,583 ) 2  0,292  PA 0,125

Kp = 0,80 La relación que existe entre las constantes de equilibrio a dos temperaturas diferentes se expresa en la ecuación de Van’t Hoff en su forma integrada, siempre y cuando la variación de entalpía estándar (Ho) sea constante en el intervalo de temperaturas en el que se está trabajando. Así, a partir de los datos de las constantes de equilibrio a 473 y 523 K obtenidas en el apartado anterior

22

29

Química para Ingeniería. Problemas y prácticas



y aplicando la ecuación de Van’t Hoff [X], se puede calcular el valor de H o para la reacción:

ln

Kp (T2 ) Kp (T1)



Ho  1 1     R  T2 T1 

[X]

donde: Kp (T2): constante de equilibrio a la temperatura T2 Kp (T1): constante de equilibrio a la temperatura T1 Ho: variación de entalpía estándar de la reacción R: constante de los gases ideales Sustituyendo los datos de las constantes Kp (473 K) = 0,07; Kp (523 K) = 0,25 y R = 1,98 cal mol-1 grado-1, se tiene: ln

0,25 H o  1 1      0,07 1,98  523 473 

Despejando se obtiene la variación de entalpía para la reacción: Ho = 12470,6 cal mol-1 A partir del valor de entalpía, aplicando de la ecuación [X] podemos calcular la temperatura a la cual ha reaccionado el 70% del reactivo A sabiendo que Kp = 0,80: ln

0,80 12470 ,6  1 1      0,07 1,98   473 

Despejando se obtiene la temperatura deseada: T = 578,9 K Por último se calcula el valor de Go. Para ello se utiliza la ecuación [VI]: Go (578,9 K) = – R·T·lnKp Go (578,9 K) = –1,98·578,9·ln 0,80 Go (578,9 K) = 255,8 cal mol-1

30

23

Capítulo 1. Termodinámica y Equilibrio químico



Para calcular el valor de So se utiliza la ecuación [VII]: Go (578,9K) = Ho (578,9 K) – T·So (578,9 K) 255,8 = -12470,6 – 578,9·So (578,9 K) So (578,9) = - 22 cal mol-1 grado-1

9. A 25 oC el valor de Kp para la siguiente reacción de equilibrio es 0,114: N2O4 (g)  2 NO2 (g) En un recipiente de un litro de capacidad se introducen 0,05 moles de tetróxido de dinitrógeno a 25 oC. Calcular las presiones parciales del tetróxido de dinitrógeno y del dióxido de nitrógeno una vez alcanzado el equilibrio. Resolución: Para calcular las presiones parciales del tetróxido de dinitrógeno y del dióxido de nitrógeno en el equilibrio se ha de aplicar la ley de Dalton: PN2O4 = xN2O4·PT PNO2 = xNO2·PT Por tanto se tienen que calcular las fracciones molares de los dos gases en el equilibrio y la presión total en el equilibrio. Para calcular las fracciones molares se necesita calcular el número de moles de N2O4 y de NO2 en el equilibrio y para ello tenemos que hacer un balance molar de la reacción a partir de la concentración inicial de N2O4 y considerando que en el equilibrio han reaccionado x moles de esta especie. N2O4 (g)

24



2 NO2 (g)

Moles iniciales

0,05

-

Moles reaccionados

x

-

Moles equilibrio

0,05-x

2x

31

Química para Ingeniería. Problemas y prácticas



Teniendo en cuenta la expresión de la constante de equilibrio en función de las concentraciones: Kc 

[NO 2 ]2 [N 2O 4 ]

se puede calcular el valor de x. Para ello se ha de conocer previamente el valor de Kc. La Kc se calcula a partir del valor de Kp que es 0,114. La relación entre Kp y Kc para cualquier reacción a una temperatura determinada viene dada por la ecuación [XI]: Kp = Kc·(R·T)n

[XI]

donde: Kp: constante de equilibrio en función de las presiones parciales Kc: constante de equilibrio en función de las concentraciones R: constante de los gases ideales (l atm grado-1 mol-1) T: temperatura (Kelvin) n: variación algebraica de los coeficientes estequiométricos de los gases Así, teniendo en cuenta que para esta reacción n = 1 se puede escribir: 0,114 = Kc·(0,082·298)1 Despejando se obtiene: Kc = 0,0047 Con el valor de Kc se calcula el valor de x y las concentraciones de las especies en el equilibrio: 2

 2x    1 0,0047     0,05  x    1  

Resolviendo la ecuación de segundo grado se obtiene que x = 0,0071 moles con lo que las concentraciones en el equilibrio de ambas especies son: [NO2]equilibrio = 0,0142 M; [N2O4]equilibrio = 0,0429 M Se puede calcular el número total de moles en el equilibrio como la suma de los de tetróxido de dinitrógeno más los de dióxido de nitrógeno: moles de N2O4 en el equilibrio = 0,0429 32

25

Capítulo 1. Termodinámica y Equilibrio químico



moles de NO2 en el equilibrio = 0,0142 moles totales en el equilibrio = 0,0142 + 0,0429 = 0,0571 Una vez conocidos los moles totales en el equilibrio podemos calcular la presión total considerando que la mezcla de gases tiene un comportamiento ideal: P·V = n·R·T P·1 = 0,0571·0,082·298 P = 1,4 atm Con el valor de la presión total en el equilibrio, los moles de N2O4 y NO2 y los moles totales se calculan las presiones parciales en el equilibrio aplicando la ley de Dalton: PN2O4 

0,0429  1,4 0,0571

PN2O4 = 1,05 atm

PNO2 

0,0142  1,4 0,0571

PNO2 = 0,35 atm

10. En un recipiente de 10 l se introducen 2 moles de A y 1 mol de B. Se calienta a 300 oC y se establece el siguiente equilibrio: A (g) + 3 B (g)  2 C (g) Sabiendo que cuando se alcanza el equilibrio el número de moles de B es igual al de C, calcular: a) Las concentraciones de cada una de las especies en el equilibrio. b) El valor de Kp y Kc a 300 oC. Resolución: a) Para calcular las concentraciones de las especies en el equilibrio se plantea el balance de moles para la reacción en las condiciones indicadas considerando

26

33

Química para Ingeniería. Problemas y prácticas



que reaccionan “x” moles de la especie A: A (g)

+

3 B (g)



2 C (g)

Moles iniciales

2

1

-

Moles reaccionados

x

3x

-

Moles equilibrio

2-x

1-3x

2x

Cuando se alcanza el equilibrio se tiene que el número de moles de la especie B es igual al número de moles de la especie C. Esto nos permite calcular el número de moles reaccionados ya que: Moles de B en equilibrio = Moles de C en equilibrio 1-3x = 2x x = 0,2 moles Con el valor de x y el volumen total del recipiente (10 l) se puede calcular la concentración de las especies en el equilibrio: [A] = (2-x)/10 = (2-0,2)/10 = 0,18 M [B] = (1-3x)/10 = (1-3·0,2)/10 = 0,04 M [C] = (2·x)/10 = (2·0,2)/10 = 0,04 M b) Con los valores de las concentraciones de las especies A, B y C y teniendo en cuenta la expresión de la constante de equilibrio Kc se puede calcular: Kc

[C ] 2 [ A ][B ]3

Kc = 138,9 Finalmente, a partir del valor de Kc, y teniendo en cuenta la temperatura (573 K) y la relación entre Kp y Kc, se puede calcular el valor de Kp: Kp = Kc·(R·T)n donde: Kp: constante de equilibrio en función de las presiones parciales Kc: constante de equilibrio en función de las concentraciones R: constante de los gases ideales T: temperatura (Kelvin) n: variación algebraica de los coeficientes estequiométricos de los gases 34

27

Capítulo 1. Termodinámica y Equilibrio químico



Así, teniendo en cuenta que para esta reacción n = -2 se puede escribir: Kp = 138,9·(0,082·573)-2 Despejando se obtiene: Kp = 0,063

11. El ácido fórmico (HCO2H) tiene una constante de disociación de 1,8·10-4 a 25 oC. Calcular las concentraciones de HCO2H, HCO2� y H3O+ en el equilibrio para una disolución de ácido fórmico 0,01 M. Resolución: La reacción ácido-base que tiene lugar es el equilibrio de disociación o ionización parcial del ácido. Puesto que la constante de acidez es pequeña, se trata de un ácido débil que estará muy poco disociado. Así se tiene el siguiente balance de concentraciones: HCO2H

+

H2O



HCO2�

+

H3O+

Concentración iniciales

0,01

-

-

Concentración reaccionada

x

-

-

Concentración final

0,01-x

x

x

Teniendo en cuenta la expresión de la constante de disociación del ácido fórmico: Ka 

[HCO 2 ]  [H 3 O  ] [HCO 2H]

1,8  10  4 

xx 0,01  x

Resolviendo la ecuación de segundo grado se obtiene que x = 0,0013 M.

28

35

Química para Ingeniería. Problemas y prácticas



De esta forma las concentraciones en el equilibrio de las distintas especies son: [HCO2H] = 0,0087 M [HCO2� ] = 0,0013 M [H3O+ ] = 0,0013 M

12. Para la siguiente reacción, a 298 K: 2 NOBr (g)  2 NO (g) + Br2 (g) �Go = 8,26 kJ y �Ho = 16,16 kJ. Calcular la constante de equilibrio a 298 K y a 598 K. Nota: Considerar que Cp = 0 Resolución: Para calcular la Kp a 298 K se puede emplear el valor de Go (298 K) aplicando la ecuación [VI] y teniendo en cuenta que R = 8,31 J mol-1 grado-1: Go (298 K) = – R·T·lnKp 8,26·1000 = – 8,31·298·lnKp Despejando se obtiene: Kp = 0,036 Para el cálculo de Kp a 598 K vamos a emplear la ecuación de Van’t Hoff [X]:

ln

Kp (T2 ) Kp(T1)



Ho  1 1     R  T2 T1 

donde: Kp (T2): constante de equilibrio a la temperatura T2 Kp (T1): constante de equilibrio a la temperatura T1 Ho: variación de entalpía estándar de la reacción R: constante de los gases ideales

36

29

Capítulo 1. Termodinámica y Equilibrio químico



Sustituyendo los datos Kp (289 K) = 0,036, Ho = 16160 J mol-1 y considerando R = 8,31 J mol-1 grado-1, se tiene: ln

K p (598 K ) 0,036



16160  1 1     8,31  598 298 

Despejando se obtiene el valor de Kp a 598 K: Kp = 0,95 Como la reacción es endotérmica (�Ho = 16,16 kJ) un aumento de la temperatura produce un incremento en la constante de equilibrio Kp.

30

37

Química para Ingeniería. Problemas y prácticas



Problemas para resolver 1. Sabiendo que la reacción de combustión del butano tiene una �Ho (298 K) = - 2877,4 kJ mol-1, determinar �Eo (298 K) para el proceso considerando que el agua se obtiene en estado líquido. Solución: Eo = -2868,7 kJ mol-1 2. Dadas las entalpías estándar de las siguientes reacciones: a) C3H8 (g) + 5 O2 (g)  3 CO2 (g) + 4 H2O (l)

Ho = - 2219,9 kJ

b) C (grafito) + O2 (g)  CO2 (g)

Ho = - 393,5 kJ

c) H2 (g) + 1/2 O2 (g)  H2O (l)

Ho = - 285,8 kJ

Calcular la entalpía estándar de la reacción de formación del propano: 3 C (grafito) + 4 H2 (g)  C3H8 (g) Solución: Hof = - 103,8 kJ mol-1 3. Calcular el calor de combustión molar del benceno. Datos: Energía de enlace (kcal mol-1): (C-C) = 58,6; (C=C) = 100,0; (C-H) = 87,3; (C=O) = 152,0; (O-H) = 110,2; (O=O) = 118,0. Energía de resonancia (kcal mol-1): (CO2) = 33; Anillo bencénico = 38. Solución: Hoc = - 760,6 kcal mol-1 4. a) Si un recipiente de 1 kg de cobre, que se encuentra a 100 oC se pone en contacto con 250 ml de agua a 27 oC, ¿cuál será la temperatura final del sistema? b) Determinar la temperatura alcanzada en dicho sistema si el recipiente fuera de aluminio en lugar de cobre. Datos: Ce (J grado-1 gramo-1): cobre = 0,385; aluminio = 0,903; agua = 4,180 d (agua) = 1g ml-1 38

31



Capítulo 1. Termodinámica y Equilibrio químico

Solución: a) T = 46,65 oC b) T = 60,84 oC 5. Determinar la �Ho (573 K) de la reacción de combustión del metano, sabiendo que �Ho (298 K) = - 790,4 kJ/mol. Datos: Cp (cal grado-1 mol-1): H2O (l) = 18,1; CH4 (g) = 9,0; CO2 (g) = 8,2; O2 (g) = 7,0 Nota: Considerar que el agua se obtiene en estado líquido. Solución: Ho (573 K) = - 765,8 kJ mol-1 6. El hidrógeno se considera uno de los combustibles alternativos más prometedor. Su combustión produce vapor de agua, y además libera gran cantidad de energía por gramo de combustible quemado. Otra característica importante es que no libera dióxido de carbono cuando se quema, por lo que su uso no contribuye al “efecto invernadero” producido por los combustibles fósiles. Si en un reactor, a presión constante y temperatura de 25 ºC, se lleva a cabo la combustión completa de 2 moles de hidrógeno con el doble del oxígeno estequiométricamente necesario: a) Escribir y ajustar la reacción que tiene lugar. b) Calcular la temperatura máxima de llama alcanzada en el reactor. c) Determinar la temperatura máxima de llama que se conseguirá si se lleva a cabo la misma combustión con el doble del aire estequiométrico. Datos a 25 ºC: �Hºf (kcal mol-1): agua (v) = - 57,8; Cp (cal grado-1 mol-1): nitrógeno (g) = 6,8; oxígeno (g) = 6,5; agua (v) = 7,1; Composición del aire: 20% O2; 80% N2 Solución: a) H2 (g) + 1/2 O2 (g) � H2O (v) b) Tmax = 5882,5 K c) Tmax = 1837,3 K

32

39

Química para Ingeniería. Problemas y prácticas



7. En un reactor térmicamente aislado, de 2000 litros de capacidad, se introducen a 25 oC, 10 moles de CH4 y un 100% de exceso de la cantidad de aire necesaria para que se produzca la combustión completa en condiciones adiabáticas. Una chispa provoca la explosión. a) Determinar la �E del proceso a 25 oC. b) Determinar la temperatura y la presión máxima de explosión alcanzadas en el reactor. c) Explicar si la temperatura máxima será mayor o menor que la calculada anteriormente, si la combustión se realiza utilizando el 100% de exceso de oxígeno puro en lugar de aire. Datos: composición del aire: 20% O2; 80% N2. Hof a 25 oC (kcal mol-1): CH4 (g) = - 17,9; CO2 (g) = - 94,1; H2O (v) = - 57,8 Cp, a 25 oC (cal grado-1 mol-1): CO2 (g) = 8,8; O2 (g) = 6,5; H2O (v) = 7,1; N2 (g) = 6,8 Solución: a) E = -1918 kcal b) Tmax = 2156,2 K; Pexplosión = 18,56 atm c) La temperatura máxima de explosión será mayor que la del apartado anterior 8. Cuando se realiza la combustión completa de un mol de un compuesto X, a presión constante y temperatura inicial de 298 K, en presencia del aire estequiométricamente necesario se alcanza una temperatura máxima de llama de 2000 K. Explicar, razonadamente, cómo variará la temperatura máxima alcanzada si: a) En lugar de aire se realiza la combustión con oxígeno puro en proporciones estequiométricas. b) La reacción de combustión del compuesto X tiene lugar en un 50 %. c) Se emplean 5 moles del compuesto X. Solución: a), b), c) Consultar la teoría

40

33

Capítulo 1. Termodinámica y Equilibrio químico



9. La industria metalúrgica obtiene el hierro a partir de la siguiente reacción a 2000 oC: Fe2O3 (s) + 3 CO (g)  2 Fe (l) + 3 CO2 (g) Los valores de las variables termodinámicas para dicho proceso a 2000 oC son Ho = - 10,0 kcal mol-1 y So = 4,8 cal mol-1 grado-1. a) Calcular Go para la reacción a 2000 oC. b) Si se lleva a cabo el proceso a las presiones que a continuación se indican en la tabla: 





1

760

760

2

7600

760

Calcular G para cada experiencia e indicar en qué sentido se produce la reacción. Solución: a) Go = - 20,90 kcal mol-1 b) G (experiencia 1) = - 20,9 kcal mol-1, proceso espontáneo en el sentido de izquierda a derecha. G (experiencia 2) = 10,19 kcal mol-1, proceso espontáneo en el sentido de derecha a izquierda 10. El pentacloruro de fósforo se disocia en tricloruro de fósforo y cloro según el siguiente equilibrio: PCl5 (g)  PCl3 (g) + Cl2 (g) En un recipiente de 2 litros de capacidad se introducen 4 moles de pentacloruro de fósforo y 2 moles de cloro a una temperatura de 300 oC. Calcular las concentraciones de las tres especies en el equilibrio teniendo en cuenta que el valor de Kp a 300 oC es de 2,21. Solución: [PCl5] = 1,917 M; [PCl3] = 0,083 M; [Cl2] = 1,083 M

34

41

Química para Ingeniería. Problemas y prácticas



11. La síntesis del amoníaco por el proceso Haber tiene lugar a 400 oC a través de la siguiente reacción: 1/2 N2 (g) + 3/2 H2 (g)  NH3 (g) -2

con una Kp = 1,3·10 y una Ho = - 53,02 kJ mol-1. Calcular a qué temperatura la Kp es 3,8·10-3 Considerar Cp = 0 Solución: T = 773 K 12. Para el siguiente equilibrio, la �Ho es 7944,7 cal: A (g)  B (g) + C (g) o

a) Determinar Kp e �G del proceso a 250 oC sabiendo que en el equilibrio, a esta temperatura, ha reaccionado el 46% de A y la presión total es de 1 atmósfera. b) Calcular Kp a 265 oC. c) ¿Cuál es la �So de la reacción a 250 oC? d) Si se desea aumentar la proporción de la sustancia B, ¿será conveniente disminuir la temperatura? Justificar la respuesta. e) Si alcanzado el equilibrio se disminuye la presión del sistema, explicar en qué sentido se desplazará el equilibrio. Nota: Considerar que �Ho e �So son independientes de la temperatura. Solución: a) Kp (250 ºC) = 0,27; Go (250 ºC) = 1355,9 cal mol-1 b) Kp (265 ºC) = 0,33 c) So = 12,6 cal mol-1 grado-1 d) Como la reacción es endotérmica, una disminución de la temperatura desplazará la reacción hacia los reactivos y, por tanto, disminuirá el rendimiento de obtención de B y de C e) Al disminuir la presión, el equilibrio se desplaza hacia donde hay mayor número de moles, es decir, hacia los productos

42

35





36



Problemas resueltos 1. Se estudia la cinética de la reacción entre el hidrógeno y el óxido de nitrógeno (II) para dar vapor de agua y nitrógeno. Experimentalmente se obtiene la velocidad inicial de la reacción (vo) para diferentes concentraciones de hidrógeno y óxido de nitrógeno (II) a 700 oC:





· 106 (M s-1)

[H2] (M)

[NO] (M)

1

0,010

0,025

2,4

2

0,005

0,025

1,2

3

0,010

0,0125

0,6

Determinar el orden y la constante de velocidad de la reacción considerada. Resolución: La ley de velocidad tiene la siguiente expresión general: v = k [H2]p [NO]q siendo: v: velocidad de la reacción k: constante de velocidad de la reacción [H2]: concentración del reactivo H2 [NO]: concentración del reactivo NO p: orden parcial respecto a H2 q: orden parcial respecto a NO Para determinar la ecuación de velocidad se tiene que calcular el valor de p, q y el valor de la constante de velocidad (k). 45 37

Química para Ingeniería. Problemas y prácticas 

Sustituyendo los datos de los tres experimentos en la ley de la velocidad se tiene: v1 = k · [0,010]p [0,025]q = 2,4·10-6 M s-1 v2 = k · [0,005]p [0,025]q = 1,2·10-6 M s-1 v3 = k · [0,010]p [0,0125]q = 0,6·10-6 M s-1 Para determinar el valor de p y q se combinan los experimentos de manera que sólo uno de los reactivos cambie de concentración. Comparando, por ejemplo, los experimentos 1 y 2 en los que la concentración de NO permanece constante se obtiene el valor de p: p q ([H2 ])1p v 1 k  ([H2 ])1  ([NO])1   v 2 k  ([H2 ])p2  ([NO]) q2 ([H2 ])p2

2,4  106 1,2  106



(0,01)p

 0,01    p (0,005)  0,005 

p

2  2p

p=1 Comparando los experimentos 1 y 3 en los que la concentración de H2 permanece constante se obtiene el valor de q: p q q v1 k  ([H2 ])1  ([NO])1 ([NO])1   v3 k  ([H2 ])p3  ([NO])3q ([NO])3q

2,4  106

0,6  106



(0,025)q

 0,025    q (0,0125)  0,0125 

q

4  2q

q=2 Así pues, la reacción es de orden uno respecto al H2, de orden 2 respecto al NO y de orden global 3. La ley de velocidad es: v = k [H2] [NO]2 46 38

Capítulo 2. Cinética química 

Teniendo en cuenta la ley de velocidad y sustituyendo los valores de la velocidad inicial y los de concentración inicial de los reactivos se puede despejar la k obteniéndose para los tres experimentos: k = 0,384 M-2 s-1 Si los valores obtenidos para k no fueran todos iguales, el valor de la constante se calcularía como la media aritmética.

2. Dada la reacción:

A (g) + B (g)

k1 k-1

C (g) + D (g)

Siendo k1 y k-1 las constantes de velocidad de la reacción directa e inversa y cuyos valores, a 50 oC, son 0,2 M-1s-1 y 4,0·10-4 M-1s-1, respectivamente. Teniendo en cuenta que al aumentar la temperatura a 90 oC el valor de k1 se duplica y el de k-1 se multiplica por 8, calcular: a) La constante de equilibrio a 50 oC y a 90 oC. b) La energía de activación para las reacciones directa e inversa. c) La Ho para la reacción. Resolución: a) Para calcular las constantes de equilibrio a 50 y a 90 oC hay que tener en cuenta que éste se alcanza como consecuencia de que las velocidades de la reacción directa y de la reacción inversa se igualan. Ambas reacciones son de orden 2 como se deduce de las unidades de la constante de velocidad (M-1s-1) por lo que se puede escribir: v d  vi k 1  [ A ]  [B ]  k 1  [C ]  [D ]

Por tanto se puede escribir: k1 [C]  [D]   K eq k 1 [ A]  [B]

47 39

Química para Ingeniería. Problemas y prácticas 

Así pues, sustituyendo los valores de k1 y k-1 a 50 oC (323 K) y a 90 oC (363 K) se pueden calcular las constantes de equilibrio a las dos temperaturas: K eq (323 K ) 

0,2

Keq (323 K) = 500

4,0  10  4

K eq (363 K ) 

0,2  2 0,0032

Keq (363 K) = 125

b) Las energías de activación de las reacciones directa en inversa se calculan a partir de la ecuación de Arrhenius [I], que relaciona dos constantes de velocidad a dos temperaturas:

ln

E  1 1 k2   a    k1 R  T2 T1 

[I]

En este caso se tienen dos constantes de velocidad para la reacción directa a dos temperaturas distintas y dos constantes de velocidad para la reacción inversa a dos temperaturas distintas: T = 323 K  k1 = 0,2 M-1s-1 T = 363 K � k1 = 0,4 M-1s-1 T = 323 K  k-1 = 4,0·10-4 M-1s-1 T = 363 K � k-1 = 3,2·10-3 M-1s-1 Aplicando la ecuación de Arrhenius (I) a los datos de la reacción directa, y teniendo en cuenta que R = 1,98 cal K-1 mol-1 se obtiene su energía de activación: ln

k 1( 363 K ) k 1( 323 K )

ln



(E a ) d R

1   1    363 323  

0,4 (E )  1 1   a d   0,2 1,98  363 323 

(Ea)d = 4022,05 cal

48 40

Capítulo 2. Cinética química 

De la misma manera, pero empleando los datos de la reacción inversa, se puede calcular su energía de activación:

ln

k 1( 363 K ) k 1( 323 K ) ln

3,2x10 3 4,0 x10

4





(E a ) i R

1   1    363 323  

(Ea )i  1 1     1,98  363 323 

(Ea)i = 12068,50 cal c) La entalpía de la reacción se puede calcular teniendo en cuenta que: Ho = (Ea)d – (Ea)i De esta forma: Ho = (Ea)d – (Ea)i = 4022,05 – 12068,50 = - 8046,45 cal

3. La constante de velocidad para la descomposición del acetaldehído se ha medido en el rango de temperaturas comprendido entre 700 y 1000 K. Los valores se muestran en la siguiente tabla:  (oC)

427

457

487

517

537

567

637

727

 (M-1s-1)

0,011

0,035

0,105

0,343

0,789

2,17

20,0

145

Determinar gráficamente la energía de activación y la constante de velocidad a 800 K a partir de los valores tabulados. Resolución: La constante de velocidad varía con la temperatura según la ecuación de Arrhenius:

k  A  eEa / RT 49 41

Química para Ingeniería. Problemas y prácticas 

donde: A: factor de frecuencia Ea: energía de activación (en julios o en calorías) R: constante de los gases ideales (en J K-1 mol-1 o cal K-1 mol-1) T: temperatura (Kelvin) Si se toman logaritmos neperianos en ambos lados de la igualdad se obtiene la siguiente ecuación:

ln k  ln A 

Ea 1  R T

que es la ecuación de una recta (y = b + mx). Si se representa ln k (y) frente a 1/T (x) se obtiene una recta cuya pendiente (m) es:

m

Ea R

[II]

De la pendiente de esta recta se puede, por tanto, calcular la energía de activación. Para ello se construye la siguiente tabla:

o

T ( C)

427

457

487

517

537

567

637

727

T (K)

700

730

760

790

810

840

910

1000

1,40

1,37

1,31

1,26

1,23

1,19

1,10

1

k (M s )

0,011

0,035

0,105

0,343

0,789

2,17

20

145

ln k

-4,51

-3,35

-2,25

-1,07

-0,23

0,77

2,99

4,98

3

-1

1/T x 10 (K ) -1 -1

50 42

Capítulo 2. Cinética química 

Al representar ln k frente a 1/T, se obtiene la siguiente recta: 6 4

ln k

2 0 -2 -4 -6

0,0010

0,0011

0,0012

0,0013

0,0014

0,0015

-1

1/T (K )

Para calcular la pendiente de la recta (y/x), se toman dos puntos relativamente alejados de la misma: m = - 23725 sustituyendo en [II] tenemos:  23725  

Ea 1,98

Ea = 46975,5 cal = 47 kcal Para determinar la constante de velocidad a 800 K se debe calcular la inversa de la temperatura

1 1   1,25  10 3 K 1 T 800 y buscar en la gráfica para este valor de abscisas el correspondiente valor de la ordenada. Así: ln k = - 0,60 k = 0,55 M-1s-1 51 43

Química para Ingeniería. Problemas y prácticas 

4. Para la siguiente reacción ajustada: 2 NO2 + O3 � N2O5 + O2 La ley de velocidad determinada experimentalmente es: v = k [NO2][O3] Explicar qué mecanismo es consistente para la reacción indicada.  i) NO2 + NO2 � N2O4

etapa lenta

ii) N2O4 + O3 � N2O5 + O2

etapa rápida



 i) NO2 + O3 � NO3 + O2

etapa lenta

ii) NO3 + NO2 � N2O5

etapa rápida

Resolución: Para que un mecanismo propuesto sea consistente con la ley de velocidad determinada experimentalmente, tienen que cumplirse dos condiciones: - La suma de todas las etapas debe de coincidir con la reacción global. - La expresión de la velocidad determinada experimentalmente debe ser coincidente con la velocidad de la etapa más lenta. A continuación se comprueba si cada uno de los mecanismos propuestos cumple estas dos condiciones.   Primero comprobamos si la suma de las dos etapas (o procesos elementales) coincide con la reacción global:

52 44

NO2

+

NO2



N2O4

N2O4

+

O3



N2O5

+

O2

2 NO2

+

O3



N2O5

+

O2

Capítulo 2. Cinética química 

Se puede observar que la suma de las dos etapas sí que da la reacción global y que el N2O4 es un intermedio de reacción. Por otra parte, la ley de velocidad para el proceso elemental más lento será: v = k [NO2]2 Como se puede observar la ecuación de velocidad no coincide con la determinada experimentalmente por lo que este mecanismo no es posible.  

 De nuevo comprobamos si la suma de las dos etapas coincide con la reacción global: NO2

+

O3



NO3

NO3

+

NO2



N2O5

2 NO2

+

O3



N2O5

+

O2

+

O2

En este caso también se obtiene la reacción global al sumar los dos procesos elementales siendo el intermedio de reacción el NO3. En este segundo mecanismo la velocidad del proceso elemental más lento se puede escribir como: v = k [NO2] [O3] Por lo tanto, la velocidad de la etapa más lenta coincide con la ley de velocidad determinada experimentalmente con lo que el mecanismo es posible.

5. Considerar que la siguiente reacción: AB tiene una �H de 20,5 kcal y una Ea de 39,7 kcal. Si se utiliza un catalizador adecuado, la energía de activación disminuye a 28,2 kcal. ¿Cual será el valor de �H para la reacción catalizada? Justificar la respuesta.

53 45

Química para Ingeniería. Problemas y prácticas 

Resolución: La presencia de un catalizador aumenta la velocidad del proceso porque es capaz de disminuir la energía de activación de la reacción. Como consecuencia muchas más moléculas de reactivos pueden transformarse en el complejo activado y dar lugar a los productos. Sin embargo, el catalizador no modifica los parámetros termodinámicos de la reacción con lo que: H = 20,5 kcal

54 46

Capítulo 2. Cinética química 

Problemas para resolver 1. Para la siguiente reacción: (CH3)3CBr + OH� � (CH3)3COH + Br � Se han llevado a cabo una serie de experimentos y se han obtenido los resultados que se indican en la tabla siguiente:







 (M s-1)

[(CH3)3CBr] (M)

[OH ] (M)

1

0,10

0,10

0,001

2

0,20

0,10

0,002

3

0,30

0,10

0,003

4

0,10

0,20

0,002

A partir de estos resultados determinar: a) La ley diferencial de velocidad del proceso. b) La constante de velocidad. Solución: a) v = k [(CH3)3CBr] [OH� ] b) k = 0,1 M-1 s-1 2. Dada la reacción: A (g) o

-5

k1 k-1

B (g)

-1

A 55 C, k1 = 1,50·10 min ; k-1 = 0,11·10-5 min-1 A 85 oC, k1 = 6,10·10-5 min-1; k-1 = 0,38·10-5 min-1 55 47

Química para Ingeniería. Problemas y prácticas 

Calcular: a) La energía de activación de las reacciones directa e inversa. b) La constante de equilibrio a 55 y 85 oC. c) La Ho para la reacción. Solución: a) Ea (directa) = 10890 cal; Ea (inversa) = 9625 cal b) Keq (55 oC) = 13,63; Keq (85 oC) = 16,05 c) Ho = 1265 cal mol-1 3. En el estudio de descomposición del pentóxido de dinitrógeno se obtuvieron los siguientes datos:  (oC)

25

35

45

55

65

 (s-1)

1,74·10�5

6,61·10�5

2,51·10�4

7,59·10�4

2,40·10�3

Determinar gráficamente: a) La energía de activación de la reacción. b) La constante de velocidad a la temperatura de 50 oC. Solución: a) Ea = 24540 cal b) k (50 oC) = 4,2·10-4 s-1 4. La descomposición del peróxido de hidrógeno según la reacción global: 2 H2O2 (ac) � 2 H2O (l) + O2 (g) está catalizada por iones yoduro (I�). Experimentalmente se encontró que la ley de velocidad es v = k [H2O2] [I� ]. ¿Cuál de los mecanismos propuestos es posible? 

56 48

Capítulo 2. Cinética química 

  2 H2O2 + I� � 2 H2O + IO2� IO2�



� I + O2

etapa lenta etapa rápida

 

 H2O2 + I� � H2O + IO� �



2 IO � I +

etapa lenta

IO2�

etapa rápida

 

 H2O2 + I� � H2O + IO� �

etapa lenta �

H2O2 + IO � H2O + O2 + I

etapa rápida

Solución: El único mecanismo posible es el III. 5. Cuando tiene lugar el siguiente proceso, a presión y temperatura constantes, se desprenden 33,4 kcal: N2O (g) + NO (g)  N2 (g) + NO2 (g) a) Si la energía de activación de la reacción inversa es de 83,3 kcal, ¿cuál será la energía de activación de la reacción directa? b) ¿Cuál será el valor de la entalpía si la velocidad del proceso se aumenta por adición de un catalizador? c) Esbozar el perfil energético de los procesos catalizado y no catalizado. Solución: a) Ea (directa) = 49,9 kcal b) Ho = - 33,4 kcal c) Consultar la teoría

57 49



&DStWXOR3UREOHPDVSURSXHVWRVGH7HUPRGLQiPLFD (TXLOLEULRTXtPLFR\&LQpWLFDTXtPLFD

50



1. En un reactor a presión constante y temperatura de 25 oC se lleva a cabo la combustión completa, en condiciones adiabáticas, de 2 moles de monóxido de nitrógeno con el doble del oxígeno estequiométrico obteniéndose dióxido de nitrógeno. a) Escribir y ajustar la reacción que tiene lugar. b) Calcular la temperatura máxima de llama alcanzada en el reactor. c) Calcular la temperatura máxima de llama cuando se lleva a cabo la misma combustión con el doble del aire estequiométrico. Datos a 25 ºC: Hof (kJ mol-1): monóxido de nitrógeno (g) = 90,4; dióxido de nitrógeno (g) = 33,3; Cp (J grado-1 mol-1): oxígeno (g) = 29,5; dióxido de nitrógeno (g) = 36,0; nitrógeno (g) = 28,0 Considerar que el aire contiene 20% de oxígeno y 80% de nitrógeno. Solución: a) NO (g) + 1/2 O2 (g) � NO2 (g) b) Tmax = 1423,1 K c) Tmax = 648,8 K 2. En un reactor se introducen a 25 oC, 5 moles de monóxido de nitrógeno y el oxígeno estequiométricamente necesario para que se produzca dióxido de nitrógeno. Calcular la temperatura máxima alcanzada en el reactor considerando que la reacción tiene lugar en un 90% sabiendo que la Hoc, a 25 oC, es 57,07 kJ mol-1 de monóxido de nitrógeno. Datos, a 25 oC: Cp (J grado-1 mol-1): monóxido de nitrógeno (g) = oxígeno (g) = 29,5; dióxido de nitrógeno (g) = 36,0 Solución: Tmax = 1691,8 K 3. Un recipiente metálico contiene 49 litros de acetileno a una atmósfera de presión y a 25 ºC. Si se produjera por completo su combustión adiabática, en presencia de la cantidad estequiométrica de aire y a presión constante, ¿se produciría la fusión del hierro? 61 51

Química para Ingeniería. Problemas y prácticas 

Datos a 25 ºC: Hºf (kcal mol-1): acetileno (g) = 54,4; dióxido de carbono (g) = -94,0; agua (v) = -57,8 Cp (cal mol-1 grado-1): dióxido de carbono (g) = 8,9; agua (v) = 8,0; nitrógeno (g) = 7,0 T fusión hierro = 1600 ºC Considerar que el aire esta formado por 20% de oxígeno y 80% de nitrógeno. Solución: Sí que se producirá la fusión del hierro porque la temperatura máxima de llama que se alcanza en la combustión es 3432 K. 4. Se introducen en un reactor 2 moles de metanol y oxígeno en exceso para llevar a cabo la combustión completa en condiciones adiabáticas. Teniendo en cuenta que la mezcla inicial se encuentra a 25 oC y que se obtiene dióxido de carbono y vapor de agua: a) Escribir y ajustar la reacción que tiene lugar. b) Determinar el número de moles de oxígeno que se tienen que introducir en el reactor si se desea que la temperatura máxima de llama sea de 5800 oC. Datos a 25 oC: Cp (cal grado-1 mol-1): CO2 (g) = 8,8; H2O (v) = 7,9; O2 (g) = 7,0 Hof (kcal mol-1): H2O (v) = - 57,8; CO2 (g) = - 94,0; CH3OH (l) = - 36,9 Solución: a) CH3OH (l) + 3/2 O2 (g) � CO2 (g) + 2 H2O (v) b) 4,5 moles de oxígeno 5. a) Calcular la temperatura máxima alcanzada cuando se quema un mol de acetileno (C2H2, g) en un recipiente cerrado de paredes rígidas de 200 litros de capacidad, térmicamente aislado, a la temperatura inicial de 25 oC, si se introduce el aire estequiométricamente necesario para que la combustión sea completa. b) ¿Cuál será la presión máxima alcanzada en el recipiente después de la explosión en dichas condiciones?

62 52

Capítulo 3. Problemas propuestos de Termodinámica, Equilibrio químico y Cinética química 

c) Explicar cómo afectará a la temperatura máxima alcanzada la utilización de oxígeno puro, en condiciones estequiométricas, en vez de aire. Datos, a 25 oC: �Hof (kJ mol-1): acetileno (g) = 226,7; dióxido de carbono (g) = - 393,5; agua (v) = - 285,8 Cp (J grado-1 mol-1): nitrógeno (g) = 28,4; dióxido de carbono (g) = 36,8; agua (v) = 29,7 Composición del aire: 20% oxígeno, 80% nitrógeno. Solución: a) Tmax = 4947,8 K b) Pmax = 26,4 atm c) La temperatura máxima será mayor 6. El etileno (C2H4, g) es un compuesto orgánico que presenta diversas aplicaciones, entre las que destaca la obtención de polietileno, uno de los plásticos más ampliamente utilizados. Además, por ser un hidrocarburo, puede ser utilizado como combustible para obtener energía. Con este fin, se introducen en un reactor de 50 litros, a la temperatura de 25 oC, 5 moles de etileno y un 20% de exceso de la cantidad de aire necesaria para su combustión completa. Una chispa provoca la explosión. Determinar la temperatura y la presión máximas alcanzadas en el reactor si consideramos que el proceso tiene lugar en condiciones adiabáticas. Datos a 25 oC: Hof (kJ mol-1): C2H4 (g) = 52,3; CO2 (g) = - 393,5; H2O (v) = - 241,8 Cp (J grado-1 mol-1): CO2 (g) = 36,8; H2O (v) = 33,2; O2 (g) = 29,4; N2 (g) = 29,1 Composición del aire (% molar): 80% nitrógeno; 20% oxígeno. Solución: Tmax = 3453,9 K; Pmax = 538,1 atm 7. La reacción de formación del amoníaco mediante el proceso Haber es la siguiente: N2 (g) + 3 H2 (g)  2 NH3 (g)

63 53

Química para Ingeniería. Problemas y prácticas 

Considerando que se hacen reaccionar 100 moles de nitrógeno y 100 moles de hidrógeno, a 400 oC, en un reactor de 50 l de capacidad y teniendo en cuenta que la presión que se alcanza en el equilibrio es de 200 atmósferas: a) Calcular la Kp del proceso a 400 oC. b) Si la reacción se lleva a cabo partiendo de una mezcla de los tres gases con PN2 = PH2 = 100 atm y PNH3 = 10 atm, predecir si ésta será espontánea o no a 400 oC. c) Calcular Kp a 600 oC considerando que HR = - 92 kJ y que Cp = 0. Considerar un comportamiento ideal de los gases. Solución: a) Kp (400 ºC) = 8,6·10-6 b) G = - 2876,6 cal; proceso espontáneo de izquierda a derecha c) Kp (600 ºC) = 2,0·10-7 8. El isooctano (C8H18) se utiliza como aditivo antidetonante de las gasolinas y se obtiene mediante una reacción catalizada a partir de dos compuestos provenientes del petróleo, el 2-metilpropano (C4H10) y el 2-metilpropeno (C4H8) según la siguiente reacción: C4H10 + C4H8  C8H18 La Keq para la reacción a 298 K es 4,3·106 y la Ho a 298 K es - 78,58 kJ a) Calcular Go a 298 K b) Determinar la Keq a 1073 K c) Calcular Go a 1073 K d) Explicar a qué temperatura se obtendrá mayor cantidad de isooctano, a 298 o a 1073 K. e) En la preparación industrial del isooctano la reacción se lleva a cabo a temperatura elevada. Dar una explicación a este hecho. f) ¿Cuál es el objetivo del catalizador? ¿Afecta a la reacción directa más que a la inversa? Justificar las respuestas. g) Dibujar el perfil energético de la reacción con los datos disponibles, en presencia y en ausencia de catalizador. Considerar Cp = 0

64 54

Capítulo 3. Problemas propuestos de Termodinámica, Equilibrio químico y Cinética química 

Solución: a)Go (298 K) = - 37,8 kJ mol-1 b) Keq (1073 K) = 4,8·10-4 c)Go (1073 K) = 68,4 kJ mol-1 d) A 298 K puesto que a esa temperatura la constante de equilibrio es mayor e) Consultar la teoría f) Consultar la teoría g) Consultar la teoría 9. En la reacción de obtención de amoníaco por el proceso Haber la Kp varía de 6,0·105 a 4,9·10-3 cuando la temperatura a la que se lleva a cabo dicha reacción aumenta de 25 ºC a 300 ºC. a) Teniendo en cuenta estos datos, indicar si se puede deducir que la reacción es exotérmica. Justificar la respuesta. b) Elegir, justificando la respuesta, una forma de aumentar el rendimiento en amoníaco. c) Si se quisiera aumentar la velocidad de la reacción sin perjudicar el rendimiento del proceso, explicar qué se debería hacer. d) ¿Qué similitudes presenta esta reacción con la de obtención del ácido sulfúrico empleada en la industria? Solución: a) Consultar la teoría. Principio de Le Chatelier b) Consultar la teoría. Principio de Le Chatelier c) Consultar la teoría d) Consultar la teoría 10.- Una de las reacciones del método de contacto empleado para la síntesis industrial del ácido sulfúrico es la reacción de oxidación del dióxido de azufre a trióxido de azufre en estado gaseoso. a) Escribir y ajustar esta reacción de equilibrio.

65 55

Química para Ingeniería. Problemas y prácticas 

b) Explicar como afectaría al rendimiento de la reacción: b1) la disminución de la temperatura. b2) la disminución de la presión. c) ¿Qué similitudes presenta esta reacción con la de obtención del amoníaco empleada en la industria? Solución: a); b; c) Consultar la teoría 11. Se tiene el siguiente equilibrio: SO3 (g)  SO2 (g) + 1/2 O2 (g) Cuando se introduce una muestra de 1,6 g de SO3 en un recipiente de 1,52 l a 900 K se encuentra que en el equilibrio hay presentes 0,38 g de SO2. a) Calcular la presión parcial y el porcentaje de cada uno de los gases en condiciones de equilibrio. b) Determinar el valor de Kc y Kp para la reacción de disociación a 900 K. Datos: Ar (O) = 16; Ar (S) = 32 Solución: a) PSO3 = 0,68 atm; PSO2 = 0,29 atm; PO2 = 0,15 atm; 60,9% de SO3; 26,1% de SO2; 13% de O2 b) Kp = 0,162; Kc = 0,019 12. Se hace reaccionar una mezcla equimolecular de NO (g) y O2 (g) a 457 K para obtener NO2 (g). Cuando se alcanza el equilibrio se observa que se ha oxidado el 60% del NO inicial y que la presión total es 2 atmósferas. Calcular Kp y Kc para el proceso a dicha temperatura. Solución: Kp = 1,65; Kc = 10,10 13. Para la reacción: (CH3)2CHOH (g)  (CH3)2CO (g) + H2 (g)

66 56

Capítulo 3. Problemas propuestos de Termodinámica, Equilibrio químico y Cinética química 

La constante de equilibrio es 0,44 a 452 K. a) ¿Cuál es el valor de Go a 452 K? b) Si en un recipiente a 452 K se introducen (CH3)2CHOH, (CH3)2CO e hidrógeno a 1,0; 0,1 y 0,2 atmósferas, respectivamente ¿será espontánea la reacción en estas condiciones? c) Si una vez alcanzado el equilibrio se aumenta la presión total del sistema, explicar en qué sentido se desplazará el equilibrio. d) Sabiendo que es una reacción exotérmica razonar si un aumento de temperatura favorecerá o no la formación de (CH3)2CO. Solución: a) Go = 734,8 cal mol-1 b) G = - 2766,4 cal; reacción espontánea en sentido de izquierda a derecha c) Consultar la teoría d) Consultar la teoría 14. En un recipiente de 0,5 litros se introduce CH4 a la presión de 100 atmósferas y a la temperatura de 1230 oC. Tras alcanzarse el equilibrio a dicha temperatura la presión parcial del hidrógeno es de 4,9 atmósferas. Sabiendo que la reacción que tiene lugar es: 2 CH4 (g)  C2H6 (g) + H2 (g) a) Calcular la Kp y la Kc para el proceso a 1230 oC. b) Determinar Kp a 500 oC. c) Indicar razonadamente hacia dónde se desplazará el equilibrio en los siguientes casos: i) se aumenta el volumen del sistema. ii) se añade un catalizador. iii) se aumenta la temperatura del sistema. Datos a 25 oC: Hof (kJ mol-1): CH4 (g) = - 74,8; C2H6 (g) = - 84,7 Considerar que la entalpía de la reacción es independiente de la temperatura.

67 57

Química para Ingeniería. Problemas y prácticas 

Solución: a) Kp = Kc = 0,003 b) Kp (500 oC) = 2,20·10-5 c) Consultar la teoría 15. Se estudia en el laboratorio la velocidad de formación de NOBr según la siguiente reacción: 2 NO (g) + Br2 (g) � 2 NOBr (g) Para ello se llevan a cabo una serie de experimentos en los que se comienza con distintas concentraciones de NO y de Br2, y se determina la velocidad inicial de la reacción, obteniéndose los siguientes resultados:



 [NO] (M)

[Br2] (M)

1

0,10

0,10

12

2

0,10

0,20

24

3

0,10

0,30

36

4

0,20

0,10

48

5

0,30

0,10

108

Determinar: a) La ley de velocidad de la reacción. b) La constante de velocidad. Solución: a) v = k [NO]2 [Br2] b) k = 12000 M-2 s-1

68 58

 (M s-1)

Capítulo 3. Problemas propuestos de Termodinámica, Equilibrio químico y Cinética química 

16. Para la reacción de descomposición del reactivo A en productos se encuentra que la constante de velocidad a 200 oC es k1 = 2,4·10-10 s-1 y a 252 oC es k2 = 1,1·10-8 s-1. Calcular la energía de activación y la constante de velocidad a 230 oC. Solución: Ea = 36215 cal; k (230 oC) = 2,4·10-9 s-1 17. En el siguiente gráfico se muestran los perfiles energéticos de dos reacciones A) y B). Indicar cuál de ellos corresponde al de una reacción exotérmica y determinar aproximadamente, para esa reacción, el valor de la entalpía y de la energía de activación. 



 (kcal)

 (kcal)

     

50

50

40

40

30

30

20

20

  

10

     

10

 

    





Solución: Consultar la teoría. H  -10 kcal; Ea  30 kcal 18. Dada la reacción: A (g) + B (g)

k1 k-1

C (g)

69 59

Química para Ingeniería. Problemas y prácticas 

k1 y k-1 son las constantes de velocidad de la reacción directa e inversa respectivamente, cuyos valores son: T = 25 oC

k1 = 0,8 M-1 s-1 k -1 = 1,6 ·10-3 s-1

T = 40 oC

k1 = 2,4 M-1 s-1 k -1 = 4,8·10-3 s-1

Calcular: a) La constante de equilibrio, Kc, a 25 oC. b) La energía de activación para las reacciones directa e inversa. c) La Ho de la reacción. d) Teniendo en cuenta que cuando la reacción directa se lleva a cabo en condiciones catalíticas la constante de velocidad puede expresarse en función de la temperatura según la ecuación: k = A ·e �9000/RT (R = 1,98 cal grado-1 mol-1) Determinar la relación entre la velocidad de la reacción directa catalizada y la no catalizada a 25 oC. Solución: a) Kc (25 oC) = 500 b) Ea (directa) = 13527,3 cal; Ea (inversa) = 13527,3 cal c) Ho = 0 d) Vcatalizada = 2,15·103·(Vno catalizada) 19. La constante de velocidad para una cierta reacción de orden 1 es de 4,60·10-4 s-1 a 350 oC. Si la energía de activación de la reacción es de 104 kJ mol-1, calcular la temperatura a la cual la constante de velocidad será 8,80·10-4 s-1. Solución: T = 643,8 K 20. Para el siguiente proceso: A (g) + B (g) � 2 C (g) + D (g) Cuando la reacción se realiza a 25 oC, se obtienen los siguientes resultados: k = 5,1·10-2 min-1; Ea = 65,5 kcal; Ho = - 83,7 kcal; Kp = 1,5·103 70 60

Capítulo 3. Problemas propuestos de Termodinámica, Equilibrio químico y Cinética química 

Experimentalmente se demuestra que al añadir una pequeña cantidad de cierta sustancia Z, la Ea es 30 kcal. a) Determinar la constante de velocidad de la reacción a 210 oC en ausencia de la sustancia Z. b) Indicar qué papel juega en el proceso la sustancia Z y explicar como afectará al valor de la Ho y de la Kp de la reacción la adición de dicha sustancia. c) Esbozar el perfil de la reacción tanto en presencia como en ausencia de Z. Nota: Considerar Ho y Ea independientes de la temperatura. Solución: a) k (210 oC) = 1,50·1017 min-1 b) Consultar la teoría c) Consultar la teoría 21. La siguiente reacción es de primer orden. 2 A (g) � B (g) + C (g) Se han determinado sus constantes de velocidad a diferentes temperaturas y se han representado los valores de ln k frente a 1/T, obteniéndose la siguiente gráfica: -6 -7 -8

ln k

-9 -10 -11 0,0029

0,0030

0,0031

0,0032

0,0033

0,0034

-1

1/T (K )

Determinar: a) La energía de activación de la reacción. b) La constante de velocidad a la temperatura de 40 oC. c) La temperatura a la cual k = 2,0·10-4 s-1. 71 61

Química para Ingeniería. Problemas y prácticas 

Solución: a) Ea = 25 kcal b) k (40 oC) = 1,12·10-4 s-1 c) T = 43,5 oC

22. En la siguiente tabla se dan los datos de la variación de la constante de velocidad con la temperatura para una reacción general del tipo: 2 A (g) � 2 B (g) + C (g)

 (oC)

0

25

45

65

100

 (s-1)

7,87·103

3,46·105

4,98·106

4,87·107

1,45·109

Determinar gráficamente: a) La energía de activación de la reacción. b) La constante de velocidad de la reacción a 80 ºC. Solución: a) Ea = 24447 cal mol-1 b) k = 3,6·108 s-1 23. Para la siguiente reacción ajustada: 2 NO2 + O3  N2O5 + O2 La ley de velocidad observada experimentalmente es: v = k [NO2][O3] ¿Qué mecanismo es consistente con esta ley de velocidad observada experimentalmente? Justificar la respuesta. Primera propuesta:

72 62

i) NO2 + NO2  N2O4

lento

ii) N2O4 + O3  N2O5 + O2

rápido

Capítulo 3. Problemas propuestos de Termodinámica, Equilibrio químico y Cinética química 

Segunda propuesta: i) NO2 + O3  NO3 + O2

lento

ii) NO3 + NO2  N2O5

rápido

Solución: La única propuesta consistente es la segunda. Consultar la teoría 24. La ley de velocidad para la siguiente reacción: 2 H2 (g) + 2 NO (g) � N2 (g) + 2 H2O (g) es v = k [H2] [NO]2. ¿Cuál de los siguientes mecanismos debe de ser descartado en base a la expresión de velocidad determinada experimentalmente?   N2 + NO � H2O + N

etapa lenta

N + NO � N2 + O

etapa rápida

O + H2 � H2O

etapa rápida



 H2 + 2 NO � N2O + H2O

etapa lenta

N2O + H2 � N2 + H2O

etapa rápida



 2 NO � N2O2

etapa rápida

N2O2 + H2 � N2O + H2O

etapa lenta

N2O + H2 � N2 + H2O

etapa rápida

Solución: Se deben descartar los mecanismos I y III. Consultar la teoría

73 63



    

64



Problemas resueltos 1. A partir de los potenciales de reducción estándar (Eº) de las siguientes especies: Eº (Na+ / Na) = - 2,71 V; Eº (Sn2+ / Sn) = - 0,16 V; Eº (F2 / F-) = 2,87 V; Eº (Ni2+ / Ni) = - 0,25 V; Eº (Ag+ / Ag) = 0,80 V; Eº (Au3+ / Au) = 1,50 V Indicar: a) ¿Qué especies pueden oxidar al níquel? b) ¿Qué especies son capaces de reducir a la plata? c) Si hay alguna especie que pueda oxidar al oro espontáneamente. Resolución Para que una reacción redox sea espontánea, en condiciones estándar, se debe cumplir que Go0. a) Si el níquel se oxida espontáneamente la semirreacción: Ni � Ni2+ + 2 econstituye el ánodo de la pila. La otra semicelda que constituye la pila debe actuar como cátodo. Para que la reacción sea espontánea el Eo > 0, por tanto EoCátodo > EoÁnodo Los pares redox que tengan un potencial estándar de reducción mayor que el par Ni2+/Ni podrán oxidar espontáneamente al níquel. Así pues, las especies que serán capaces de oxidar espontáneamente al Ni son: Sn2+, F2, Ag+ y Au3+.

77 65

Química para Ingeniería. Problemas y prácticas 

Según el razonamiento anterior las reacciones redox que ocurrirían espontáneamente en cada caso son las siguientes: Ni

� Ni2+ + 2 e-

Sn2+ + 2 e- � Sn Ni

+ Sn2+ � Ni2+ + Sn Ni

F2 + 2 e-

� Ni2+ + 2 e� 2 F-

Ni + F2 � Ni2+ + 2 FNi

� Ni2+ + 2 e-

2·(Ag+ + e- � Ag ) Ni + 2 Ag+ � Ni2+ + 2 Ag 3·(Ni

� Ni2+ + 2 e-)

2·(Au3+ + 3 e- � Au) 3 Ni + 2 Au3+ � 3 Ni2+ + 2 Au

b) Si la plata se reduce espontáneamente la semirreacción: Ag+ + e- � Ag constituye el cátodo de la pila. La otra semicelda que constituye la pila debe actuar como ánodo. Para que la reacción sea espontánea el Eo>0, por tanto EoCátodo > EoÁnodo Los pares redox que tengan un potencial estándar de reducción menor que el par Ag+/Ag podrán reducir espontáneamente a la plata. Así pues, las especies que serán capaces de reducir espontáneamente a la plata son: Na, Sn y Ni.

78 66

Capítulo 4. Reacciones de Oxidación y Reducción

Según el razonamiento anterior las reacciones redox que ocurrirían espontáneamente en cada caso son las siguientes: Ag+ + e-

� Ag

� Na+ + e-

Na

Ag+ + Na � Ag + Na+ 2·(Ag+ + e-

� Ag)

Sn

2+



Sn

+ 2 e-

2 Ag+ + Sn � 2 Ag + Sn2+ 2·(Ag+ + eNi



Ni

2+

� Ag) + 2 e-

2 Ag+ + Ni � 2 Ag + Ni2+ c) Si el oro se oxida espontáneamente la semirreacción: Au � Au3+ + 3 econstituye el ánodo de la pila. La otra semicelda que constituye la pila debe actuar como cátodo. Para que la reacción sea espontánea el Eo>0, por tanto EoCátodo > EoÁnodo. El par redox que tenga un potencial estándar mayor que el par Au3+/Au podrá oxidar espontáneamente al oro. Así pues, la única especie en este caso que será capaz de oxidar espontáneamente al oro es el F2. Teniendo en cuenta el razonamiento anterior la reacción redox que ocurrirá espontáneamente es la siguiente: 2·(Au 3·(F2

� Au3+ + 3e-) + 2 e-

� 2 F -)

2 Au + 3 F2 � 2 Au3+ + 6 F 79 67

Química para Ingeniería. Problemas y prácticas 

2. Considérese la siguiente celda galvánica representada esquemáticamente por: Al (s) | Al3+ (0,15 M) || Fe3+ (0,35 M), Fe2+ (0,25 M) | Pt (s) a) Indicar justificando la respuesta, si es espontáneo o no el proceso redox representado. b) Dibujar un boceto del dispositivo completo de la pila. c) Calcular el potencial de la celda. Datos: Eº (Al3+ /Al) = - 1,66 V; Eº (Fe3+ / Fe2+) = 0,77 V. Resolución a) Puesto que las concentraciones de las especies que constituyen la celda no son 1 M, las condiciones no son las estándar. Por tanto, para que la reacción sea espontánea se debe cumplir que G0. La notación de la pila indica que: - La semicelda que actúa como ánodo está formada por un electrodo de Al (s) sumergido en una disolución acuosa de Al3+ (0,15 M). Por tanto la semirreacción que ocurre es: Al � Al3+ + 3 e- La semicelda que actúa como cátodo está formada por un electrodo de platino sumergido en una disolución acuosa de Fe3+ (0,35 M) y Fe2+ (0,25 M). Por tanto la semirreacción que ocurre es: Fe3+ + e- � Fe2+

80 68

Capítulo 4. Reacciones de Oxidación y Reducción

Teniendo en cuenta que E = ECátodo – EÁnodo, la reacción será espontánea si ECátodo > EÁnodo. Para calcular ECátodo y EÁnodo se debe aplicar la ecuación de Nernst a cada semicelda. Para ello hay que escribir las semirreacciones en el sentido de reducción: Semicelda de Al3+ / Al:

Al3+ + 3 e- � Al

E (Al3+ / Al) = Eo (Al3+ / Al) –

E (Al3+ / Al) = -1,66 –

1 0,059 ·log 3 n Al

 

1 0,059 ·log = -1,68 V 3 0,15

Semicelda de Fe3+ / Fe2+: Fe3+ + e- � Fe2+ E (Fe3+ / Fe2+) = Eo (Fe3+ / Fe2+) –

E (Fe3+ / Fe2+) = 0,77 –

 

0,059 Fe 2 ·log n Fe 3

 

0,059 0,25 ·log = 0,78 V 0,35 1

Puesto que E (Fe3+ / Fe2+) > E (Al3+ / Al), la semicelda de Al3+/Al es el ánodo y la semicelda de Fe3+/Fe2+ es el cátodo. Por tanto, el potencial de la celda es E>0, lo que implica que G < 0, por tanto el proceso redox que indica la notación de la pila es espontáneo. b) El boceto del dispositivo completo de la pila es el siguiente: e-

(-) ÁNODO (-) Al

Al3+ (0,15 M)

Voltímetro Puente salino (+) Pt

CÁTODO (+)

Fe3+ (0,35 M) Fe2+ (0,25 M)

Aniones Cationes

81 69

Química para Ingeniería. Problemas y prácticas 

c) Para calcular el potencial de la pila se utiliza la expresión: E = ECátodo – EÁnodo E = E (Fe3+ / Fe2+) – E (Al3+ / Al) = 0,78 – (- 1,68) E = 2,46 V

3. Una celda galvánica está formada por una semicelda que está constituida por un electrodo de hidrógeno (P = 1 atm) sumergido en una disolución de ácido con una concentración de protones 0,5 M y otra semicelda constituida por un electrodo de plata sumergido en una disolución de nitrato de plata 0,15 M. a) Escribir las semirreacciones de oxidación, reducción y la reacción global de la pila. b) Calcular la fuerza electromotriz. c) Dibujar un boceto del dispositivo completo de la celda. d) Explicar qué efecto provocará en el valor del potencial de la pila un aumento de la presión del H2. e) Calcular la constante de equilibrio del proceso redox. f) Escribir la reacción redox que se producirá espontáneamente si la semicelda de la plata se sustituyera por un electrodo de zinc sumergido en una disolución de sulfato de zinc 0,5 M. Datos: Eº (Ag+ / Ag) = 0,80 V; Eº (Zn2+ / Zn) = - 0,76 V Resolución a) En primer lugar, debemos deducir qué semicelda actúa como ánodo y cuál como cátodo. Para ello, se deben escribir y ajustar las semirreacciones en el sentido de reducción y calcular el potencial de semicelda en las condiciones del problema, que no son las condiciones estándar, empleando la ecuación de Nernst.

82 70

Capítulo 4. Reacciones de Oxidación y Reducción

2 H+ + 2 e- � H 2 E (H+ / H2) = Eo (H+ / H2) –

E (H+ / H2) = 0 –

P 0,059 ·log H2 2 n H

0,059 ·log 2

 

1

0.5  2

= - 0,018 V

Ag+ + e- � Ag E (Ag+ / Ag) = Eo (Ag+ / Ag) –

E (Ag+ / Ag) = 0,80 –

0,059 ·log n

1

 Ag  

0,059 1 ·log = 0,75 V 1 0,15

Como E (Ag+ / Ag) >E (H+ / H2), la semicelda constituida por Ag+/Ag actuará como cátodo y la semicelda constituida por H+/H2 actuará como ánodo. Las semirreacciones de oxidación-reducción y la reacción redox global son las siguientes: 2·(Ag+ + e- � Ag) H2 � 2 H+ + 2 e2 Ag+ + H2 � 2 Ag + 2 H+ La reacción global se obtiene sumando las dos semirreacciones teniendo en cuenta que el número de electrones cedidos en el ánodo deben coincidir con los aceptados en el cátodo. b) Sabiendo que E = ECátodo – EÁnodo y empleando los potenciales de semiceldas calculados en el apartado a): E = 0,75 – (- 0,018) = 0,768 V E = 0,77 V

83 71

Química para Ingeniería. Problemas y prácticas 

c) El boceto del dispositivo completo de la pila es el siguiente:

Voltímetro

eÁNODO (-)

CÁTODO (+)

Puente salino Ag

H (g) 2 1 atm

Ag+ (0,15 M)

H+ (0,5 M)

Aniones Cationes

d) Aplicando la ecuación de Nernst a la reacción redox global: o E = E –

0,059 ·log n

H   Ag  ·PH2 



2

2

Si la presión del hidrógeno aumenta, el término que resta a Eo será más pequeño, por tanto el E de la pila será mayor. e) Para calcular la constante de equilibrio del proceso redox hay que tener en cuenta que en el equilibrio G=0 y por tanto E=0. Así pues, de la ecuación de Nernst: E = Eo –

0,059 ·log n

0 = Eo –





2

0,059 log K n

log K =

84 72

H   Ag  ·PH2

�E o ·n 0,059

2

Capítulo 4. Reacciones de Oxidación y Reducción

Sustituyendo los valores y despejando: log K =

0,80·2 0,059

K = 1,3·1027

En general, las constantes de equilibrio de las reacciones redox son muy elevadas. Este es el motivo por el que los equilibrios de oxidación-reducción están muy desplazados hacia la derecha, hacia los productos, y las reacciones redox no se suelen escribir con la doble flecha de equilibrio, sino sólo con una flecha. f) En este caso, vamos a suponer que al igual que en el apartado a) la semicelda de cinc actúa como cátodo y la semicelda del hidrógeno como ánodo. Para confirmar esta hipótesis aplicamos la ecuación de Nernst a la reacción redox global. H2 � 2 H+ + 2 eZn2+ + 2 e- � Zn H2 + Zn2+ � 2 H+ + Zn





o

E = E –

0,059 n

H  · log Zn ·PH2 

2

2

Eo = EoCátodo – EoÁnodo = Eo (Zn2+/Zn) – Eo (H+/H2) = - 0,76 – 0= - 0,76 V E = - 0,76 –

0,059 2

log

0,5  2 0,5·1

= - 0,75 V

Para que la reacción sea espontánea se debe cumplir que G0. Como E del proceso redox que hemos considerado es negativo esto implica que la reacción espontánea ocurre en el sentido contrario al que hemos supuesto. Por tanto, la reacción global espontánea es: 2 H+ + Zn � H2 + Zn2+

85 73

Química para Ingeniería. Problemas y prácticas 

4. Una pila está constituida por las siguientes semiceldas conectadas por un puente salino de cloruro potásico. Los electrodos son de platino y se unen por un cable conductor. Semicelda I: [IO3- ] = 0,01 M; [I2] = 0,50 M y [H+] = 10-3 M. Semicelda II: [IO3- ] = 0,10 M; [I2] = 0,20 M y [H+] = 10-1 M. Determinar, justificando las respuestas: a) Qué semicelda actúa como cátodo y cuál como ánodo. b) La polaridad de los electrodos. c) La fuerza electromotriz de la pila. d) La constante de equilibrio. Dato: Eº (IO3- / I2) = 1,19 V. Resolución a) Para deducir qué semicelda actua como ánodo y cúal como cátodo, se deben escribir y ajustar las semirreacciones en el sentido de reducción y calcular el potencial de semicelda en las condiciones del problema, que no son las condiciones estándar, empleando la ecuación de Nernst. Para ajustar este tipo de reacciones redox se emplea el método de ajuste del ión-electrón. Semicelda I: 2 IO3- + 12 H+ + 10 e- � I2 + 6 H2O

EI = Eo (IO3- / I2) –

EI = 1,19 –

86 74

0,059 10

0,059 n

· log

· log

 I2 

 IO  ·  H  3



0,5

0,01

2



· 10 3

2



12

 12

= 0,956 V

Capítulo 4. Reacciones de Oxidación y Reducción

Semicelda II:

2 IO3- + 12 H+ + 10 e- � I2 + 6 H2O

EII = Eo (IO3- / I2) –

EII = 1,19 –

0,059 10

0,059 n

·log

· log

 I2 

 IO  ·  H  3

0,2

0,1

2



· 10 1

 2



12

12

= 1,112 V

Como EII > EI, la semicelda II actuará como cátodo y la semicelda I como ánodo. b) El ánodo es el electrodo negativo y el cátodo el electrodo positivo. c) Sabiendo que E = ECátodo - EÁnodo y empleando los potenciales de semicelda calculados en el apartado a): E = 1,112 – 0,956 E = 0,156 V d) El cálculo de la constante de equilibrio se realiza a partir de la ecuación de Nersnt. En el equilibrio G=0 y por tanto E=0. Así pues, 0 = Eo –

0,059 · log K n

log K =

�E o ·n 0,059

Sustituyendo los valores: Eo = EoCátodo- EoÁnodo = Eo (IO3-/I2) – Eo (IO3-/I2) = 1,19 – 1,19 = 0 V log K =

0 ·10 =0 0,059

K=1

87 75

Química para Ingeniería. Problemas y prácticas 

La constante de equilibrio de una reacción redox tiene el valor particular de K = 1 sólo en el caso de que se trate de una pila de concentración, ya que en este caso el Eo vale cero.

5. Una fábrica de puertas de aluminio tiene una producción de 100 puertas por día empleando para cada puerta 5 kg de aluminio. Éste se obtiene en una fábrica adyacente por electrólisis de la bauxita (Al2O3). a) Calcular el tiempo que debe estar operando la celda electrolítica que trabaja a 2·105 A para proporcionar el aluminio necesario para abastecer a la fábrica de puertas cada día. Datos: Ar (Al)= 27; F = 96500 C/mol. Resolución a) La reacción que tiene lugar en el cátodo es la reducción del Al 3+ : Al3+ + 3 e- � Al La carga eléctrica (Q, culombios) que pasa por la celda electrolítica se relaciona con la intensidad de corriente (I, amperios) y con el tiempo (t, segundos) que ésta se aplica, según la expresión: Q = I · t Teniendo en cuenta que 1 mol de electrones genera una carga eléctrica de 96500 C, es necesario calcular los moles de electrones que pasan por la celda electrolítica para producir el aluminio necesario. La estequiometría de la reacción nos permite relacionar los moles de electrones (ne-) con los moles de Al (nAl): ne- = 3·nAl Para calcular nAl se debe calcular la masa de aluminio (mAl) necesaria para producir las puertas y relacionarla con la Ar (Al) según: nAl =

m Al A r (Al)

Los gramos de aluminio necesarios para producir 100 puertas son: mAl = 100 puertas·5000 g/puerta = 5·105 g de aluminio

88 76

Capítulo 4. Reacciones de Oxidación y Reducción

5

nAl =

5·10 27

= 18518,5 moles de Al

Por lo tanto: ne- = 3·nAl = 3 · 18518,5 = 55555,5 moles de electrones Por tanto la carga eléctrica necesaria es: 1 mol e- ----------------- 96500 C 55555,5 moles e- ----- Q Q = 96500  55555,5 = 5,36·109 C Sabiendo que Q = I · t, se puede calcular el tiempo necesario que debe estar operando esta celda electrolítica: t=

Q I

t=

5,36·10 9 2,0·10

5

= 26800 s .

1h 3600 s

t = 7,4 h

6. Se desea platear un objeto metálico. Para ello se dispone de plata metálica y de una disolución que contiene iones plata. a) Indicar qué tipo de celda se debe utilizar para realizar el proceso y dibujar con detalle cómo se han de disponer los elementos de la misma. b) Si la superficie del objeto es de 400 cm2 y el espesor que se desea obtener de 0,2 mm, calcular la intensidad de corriente que se debe utilizar si se desea recubrir el objeto en 20 min. Datos: d (plata)=10,54 g/ml; Ar (plata) = 107,9 Resolución a) Se tiene que emplear una celda electrolítica en la que el objeto metálico debe actuar como cátodo. La reacción que tendrá lugar en el cátodo es la reducción de la Ag+ a Ag. El ánodo está formado por la lámina de plata que es el metal utilizado para recubrir el objeto. 89 77

Química para Ingeniería. Problemas y prácticas 

Ag+ + e- � Ag (cátodo, reducción) Ag � Ag+ + e- (ánodo, oxidación) No se produce ninguna reacción neta sino que la plata del ánodo se oxida a Ag+ que es utilizada en el cátodo para generar el recubrimiento de Ag. Se produce un movimiento del metal Ag de un electrodo al otro (del ánodo al cátodo). Un boceto de la celda electrolítica se muestra a continuación: e-

- + Batería

Ag

Cátodo (-)

Objeto metálico

Ánodo (+) Á

Ag+

Ag+

b) Para calcular la intensidad de corriente necesaria se emplea la expresión: Q=I·t Para conocer la carga eléctrica que debe circular por la celda electrolítica es necesario calcular los moles de electrones de la siguiente semirreacción: Ag+ + e- � Ag (cátodo) En base a la estequiometría de la reacción se calculan los moles de electrones, que coinciden con los moles de Ag necesarios para recubrir el objeto. Para calcular los gramos de plata (mAg) necesarios para recubrir el objeto, calculamos primero el volumen del objeto a recubrir: S = 400 cm2 espesor = 0,2 mm = 0,02 cm Vobjeto a recubrir = 400·0,02 = 8 cm3 = 8 ml

90 78

Capítulo 4. Reacciones de Oxidación y Reducción

Puesto que la densidad de la plata es 10,54 g/ml, calculamos ahora la masa de plata necesaria para recubrir el objeto: mAg = 10,54·8 = 84,32 g de Ag El número de moles de Ag es: nAg =

nAg =

m Ag A r (Ag)

84,32 107,9

= 0,78 moles de Ag

Así pues, el número de moles de electrones es: ne- = nAg = 0,78 moles de electrones Teniendo en cuenta que 1 mol de electrones genera una carga eléctrica de 96500 C, se calcula la carga eléctrica necesaria para recubrir el objeto (Q). 96500 C

------------------ 1 mol de e -

Q

----------------- 0,78 moles de eQ = 75270 C

Aplicando la expresión Q = I · t I=

Q 75270 = t (20·60) I = 62,7 A

7. ¿Cuántos gramos de cobalto se depositarán en el cátodo de una celda electrolítica que contiene una disolución acuosa de sulfato de cobalto (II), sabiendo que el volumen de oxígeno desprendido en el ánodo es de 150 ml medidos a 25 ºC y 1 atmósfera de presión? Datos: Ar (Co) = 58,9

91 79

Química para Ingeniería. Problemas y prácticas 

Resolución Las reacciones ajustadas que tienen lugar en los electrodos de la celda electrolítica son: Co2+ + 2 e- � Co (cátodo) 2 H2O � O2 + 4 H+ + 4 e- (ánodo) A partir de los datos se calculan los moles de O2 desprendidos en el ánodo: P·V = n·R·T nO2 =

(1·150·103 ) = 6,14·10-3 moles de O2 (0,082·298)

Teniendo en cuenta la estequiometría de la reacción que tiene lugar en el ánodo, se calculan los moles de e- que circulan por la celda electrolítica: 1 mol O2 –--- 4 mol e6,14·10-3 moles de O2----- x x = 0,025 moles de eEn base a la estequiometría de la reacción que tiene lugar en el cátodo, se calculan los moles de cobalto que se obtienen al circular 0,025 moles de electrones: 1 mol Co ------ 2 mol de ey ------- 0,025 mol de ey = 0,0125 moles de Co mCo = Ar (Co) · nCo = 58,9 · 0,0125 mCo = 0,7 g de Co

92 80

Capítulo 4. Reacciones de Oxidación y Reducción

Problemas para resolver 1. Dados los valores de los potenciales de reducción de las siguientes especies: Eº (Fe2+ / Fe) = - 0,44 V; Eº (MnO4- / Mn2+) = 1,49 V; Eº (Mg2+ / Mg) = - 2,34 V; Eº (Al3+ / Al) = - 1,66 V a) Indicar qué especie es el agente oxidante más fuerte. b) Indicar qué especie es el agente reductor más fuerte. c) ¿Cuál de los metales Fe ó Mg quedará recubierto de aluminio cuando se introduce en una disolución 1 M de AlCl3? d) Si una disolución de MgSO4 1 M se guarda en un recipiente de hierro, ¿se producirá la oxidación del metal? Solución: a) MnO4- es el agente oxidante más fuerte b) Mg es el agente reductor más fuerte c) Mg d) No se observará la oxidación del metal 2. Una pila está formada por un electrodo de cinc sumergido en una disolución de sulfato de cinc de concentración 1 M y por un electrodo de hierro sumergido en una disolución de sulfato de hierro (II) de concentración 1 M. Ambas disoluciones se unen por un puente salino y se conectan mediante un cable eléctrico a un voltímetro. a) Indicar el ánodo y el cátodo de la pila, así como la polaridad de cada electrodo. b) Escribir la reacción global espontánea de la pila. c) Calcular la diferencia de potencial de la pila. d) Indicar el sentido del flujo de electrones. Datos: Eº (Zn2+ / Zn) = - 0,76 V; Eº (Fe2+ / Fe) = - 0,44 V

93 81

Química para Ingeniería. Problemas y prácticas 

Solución: a) Ánodo (-): semicelda de Zn; cátodo (+): semicelda de Fe b) Zn + Fe 2+  Zn 2+ + Fe c) �Eº = 0,32 V d) Los electrones se desplazan desde el electrodo donde ocurre la oxidación (ánodo) al electrodo donde se produce la reducción (cátodo) 3. Considere los siguientes procesos redox que tienen lugar en condiciones estándar: Cu (s) | Cu2+ (1 M) || Zn2+ (1 M) | Zn (s) Pt (s) | Cr2+ (1 M), Cr3+ (1 M) || Ag+ (1 M) | Ag (s) a) Indicar, justificando la respuesta, si son espontáneos o no los procesos redox anteriores. b) Ajustar las semirreacciones y la reacción global que tienen lugar espontáneamente. Datos, a 25 ºC: Eº (Cu2+ / Cu) = 0,34 V; Eº (Zn2+ / Zn) = - 0,76 V; Eº (Cr3+ / Cr2+) = - 0,41 V; Eº (Ag+ / Ag) = 0,80 V Solución: a) Consultar la teoría b) Consultar la teoría 4. Una celda galvánica está formada por una semicelda I donde una tira de cobre metálico está sumergida en una disolución de Cu2+ 0,18 M y por otra semicelda II en la que una tira de platino está sumergida en una disolución de Fe3+ 0,33 M y Fe2+ 0,27 M. a) Escribir la reacción que se produce en cada semicelda y la reacción global. b) Determinar el potencial de la pila. c) Dibujar un esquema detallado de la pila indicando qué semicelda actúa como ánodo y cuál como cátodo. Indicar cuál es el polo positivo y cuál el polo negativo de la pila. d) Calcular la constante de equilibrio del proceso redox. 94 82

Capítulo 4. Reacciones de Oxidación y Reducción

e) Explicar cómo se modificará el potencial de la pila si la concentración inicial de Cu2+ aumenta. Datos, a 25 ºC: Eº (Cu2+ / Cu) = 0,34 V; Eº (Fe3+ / Fe2+) = 0,77 V Solución: a) Semicelda I: Cu  Cu 2+ + 2 e- (Oxidación) Semicelda II: Fe 3+ + 1 e-  Fe 2+ (Reducción) Reacción global: Cu + 2 Fe 3+  Cu 2+ + 2 Fe 2+ b) E = 0,46 V c) Semicelda I: Ánodo (polo negativo) Semicelda II: Cátodo (polo positivo) Consultar teoría d) K = 3,8·1014 e) El potencial de la pila disminuirá 5. Se dispone de una pila formada por: Semicelda A: Electrodo de hierro sumergido en una disolución de cloruro de hierro (II) 0,1 M. Semicelda B: Electrodo de hidrógeno (presión de hidrógeno = 1 atm) sumergido en una disolución que contiene protones a la concentración 10-2 M. a) Escribir y ajustar la reacción que tiene lugar espontáneamente, justificando la respuesta. b) Determinar el potencial de la pila. c) Explicar el efecto que se producirá en el potencial de la pila si la presión del gas hidrógeno es de 2 atmósferas. d) Escribir y ajustar la reacción que tiene lugar espontáneamente si en lugar de cloruro de hierro (II) se emplea cloruro de cobre (II), justificando la respuesta. Datos: Eº (ión hierro (II) / hierro) = - 0,44 V; Eº (ión cobre (II) / cobre) = 0,34 V. Solución: a) 2 H+ + Fe  H2 + Fe 2+ b) E = 0,35 V

95 83

Química para Ingeniería. Problemas y prácticas 

c) El potencial de la pila disminuirá. Consultar la teoría d) H2 + Cu 2+  2H+ + Cu 6. Se desea recubrir de oro un objeto metálico. a) Determinar la cantidad de oro que se obtiene al pasar una corriente de intensidad 2,0 amperios durante 5 minutos por una celda electrolítica que contiene una disolución acuosa de una sal de oro (III). b) Si la disolución electrolizada fuese de una sal de plata y se hiciese funcionar durante el mismo tiempo y a la misma intensidad, ¿cuántos gramos de plata se obtendrían? Datos: Ar (plata) = 107,9; Ar (oro) = 196,9 Solución: a) 0,4 g de oro b) 0,7 g de plata 7. Una celda electrolítica en la que se produce la electrólisis del agua funciona con una intensidad de corriente de 1,5 mA durante dos horas. a) Escribir las semirreacciones que tienen lugar en cada electrodo indicando el ánodo y el cátodo de la celda. b) Escribir la reacción global. c) Calcular el volumen de O2 y de H2 obtenidos cuando se trabaja a 25 ºC y 1 atmósfera de presión. d) Calcular la cantidad de agua consumida en el proceso. Datos: Ar (O) = 16; Ar (H) = 1 Solución: a) Cátodo: 2H+ + 2 e-  H2 Ánodo: 2 H2O  O2 + 4 H+ + 4 eb) 2 H2O  O2 + 2 H2 c) V (O2) = 6,8·10-4 l; V (H2) = 1,37·10-3 l d) 1,0·10-3 g de H2O

96 84

Capítulo 4. Reacciones de Oxidación y Reducción

8. El recubrimiento con cromo (Cr) se realiza por electrólisis sumergiendo los objetos en una disolución de dicromato potásico (K2Cr2O7) en medio ácido. a) Escribir y ajustar la semirreacción del proceso mencionado. b) ¿Cuántas horas se tardará en aplicar un recubrimiento de cromo de 10-2 mm de espesor al parachoques de un automóvil cuya área superficial es de 0,25 m2 en una celda electrolítica por la que pasa una corriente de 25 A? Datos: Densidad del cromo = 7,2 g/cm3; Ar (Cr) = 52; F = 96500 C/mol Solución: a) Cr2O7 2- + 14 H+ + 12 e-  2 Cr + 7 H2O b) 2,2 h

97 85



          

86



1. Escribir y ajustar las semirreacciones y la reacción global que tienen lugar espontáneamente y en condiciones estándar cuando se ponen en contacto: a) Una disolución acuosa de KMnO4 con una lámina de zinc en medio ácido. b) Una disolución acuosa de KIO3 con una lámina de cobre en medio ácido. c) Una disolución acuosa de K2Cr2O7 con una disolución de KBr en medio ácido. Datos: Eº (MnO4- / Mn2+) = 1,49 V; Eº (Zn2+ / Zn) = - 0,76 V; Eº (IO3- / I2) = 1,19 V; Eº (Cu2+ / Cu) = 0,34 V; Eº (Cr2O72- / Cr3+) = 1,33 V; Eº (Br2 / Br-) = 1,09 V Solución: a), b), c) Consultar la teoría 2. Considere los siguientes procesos redox que tienen lugar en condiciones estándar: Pt (s) | H2O, H2O2 || Fe3+, Fe2+ | Pt (s) Pt (s) | Cr3+, Cr2O72- || H2O2, H2O | Pt (s) a) Indicar, justificando la respuesta, si son espontáneos o no los procesos redox anteriores. b) Ajustar las semirreacciones y la reacción global que tienen lugar espontáneamente. Datos: Eº (H2O2 / H2O) = 1,78 V; Eº (Fe3+ / Fe2+) = 0,77 V; Eº (Cr2O72- / Cr3+) = 1,33 V Solución: a) El primer proceso no es espontáneo; el segundo sí lo es b) Consultar la teoría 3. Una celda galvánica está constituida por dos semiceldas. Una de ellas consta de un tira de cobre sumergida en una disolución que contiene Cu2+ en concentración 1 M, mientras que la otra semicelda es un electrodo de hidrógeno (P = 1 atm) sumergido en una disolución ácida con una concentración de protones 1 M. a) Indicar el ánodo y el cátodo de la pila, así como la polaridad de cada electrodo. 101 87

Química para Ingeniería. Problemas y prácticas 

b) Escribir la reacción global espontánea de la pila. c) Escribir la representación esquemática de la celda. d) Calcular la diferencia de potencial de la pila. Datos, a 25 ºC: Eº (Cu2+ / Cu) = 0,34 V Solución: a) Consultar la teoría b) Consultar la teoría c) Consultar la teoría d) Eº = 0,34 V 4. Se desea diseñar una celda galvánica para lo cual se dispone de los siguientes elementos: - Una disolución de Cr2+ (0,10 M) y Cr3+ (0,90 M). - Una disolución de Ag+ (0,15 M). - Una lámina de plata y otra de platino. - Un puente salino. - Un voltímetro. - Cable eléctrico. a) Explicar cómo se puede averiguar cuál será el cátodo y el ánodo de la pila. b) Escribir la reacción redox que se producirá de forma espontánea y especificar la polaridad de cada electrodo. c) Determinar el potencial de la celda. d) Dibujar un boceto del dispositivo completo de la pila, indicando todos los componentes de la misma. e) Explicar qué efecto provocará en el valor del potencial de la pila el que la concentración de Ag+ sea 1 M en vez de 0,15 M. Datos: Eº (Ag+ / Ag) = 0,80 V; Eº (Cr3+ / Cr2+) = - 0,41 V Solución: a) Consultar teoría

102 88

Capítulo 5. Problemas propuestos de Reacciones de Oxidación y Reducción

b) Cátodo (positivo): Ag + + 1 e-  Ag Ánodo (negativo): Cr 2+  Cr 3+ + 1 eReacción global: Ag + + Cr 2+  Ag + Cr 3+ c) E = 1,11 V d) Consultar teoría e) El potencial de la pila aumentará 5. Una celda galvánica está constituida por dos semiceldas. La primera consta de un electrodo de platino y una disolución que contiene Fe3+ en concentración 0,2 M y Fe2+ en concentración 0,1 M, mientras que la segunda es un electrodo de hidrógeno (P = 1 atm) sumergido en una disolución ácida con una concentración de protones 0,1 M. Determinar: a) Las semireacciones de oxidación y reducción y la reacción global. b) Quien actúa como ánodo y como cátodo. c) Cuál es el polo positivo y el polo negativo de la celda. d) Cuál es el potencial de la pila. e) La constante de equilibrio de la reacción. Datos: Eº (Fe3+ / Fe2+) = 0,77 V Solución: a) Oxidación: H2  2 H+ + 2 eReducción: Fe 3+ + 1 e-  Fe 2+ Reacción global: H2 + 2 Fe 3+  2 H+ + 2 Fe 2+ b) Ánodo: Electrodo de hidrógeno. Cátodo: Semicelda Fe 3+ / Fe 2+ c) Ánodo (negativo). Cátodo (positivo) d) E = 0,85 V e) K = 1,26·1026 6. Una celda galvánica está formada por las siguientes semiceldas: Semicelda I: Una disolución de Fe2+ (0,25 M) y Fe3+ (0,35 M) en contacto con un electrodo de Pt.

103 89

Química para Ingeniería. Problemas y prácticas 

Semicelda II: Una disolución de Cu2+ (0,15 M) en cuyo interior se sumerge un electrodo de cobre. a) Indicar qué semicelda es el cátodo y cuál es el ánodo de la pila. b) Determinar el potencial de la celda. c) Dibujar un boceto del dispositivo completo de la pila. d) Explicar qué efecto provocará en el valor del potencial el que la concentración de Cu2+ fuese 1 M en vez de 0,15 M. Datos: Eº (Cu2+ / Cu) = 0,34 V; Eº (Fe3+ / Fe2+) = 0,77 V Solución: a) Semicelda I: Cátodo; Semicelda II: Ánodo b)E = 0,46 V c) Consultar teoría d) El potencial de la pila disminuirá 7. Se tiene la siguiente celda galvánica representada esquemáticamente: Mg(s) | Mg2+ (x M) || Co3+ (0,1 M), Co2+ (0,6 M) | Pt(s) a) Escribir y ajustar la reacción que se producirá en cada semicelda y la reacción global. b) Determinar la concentración de Mg2+ para obtener un potencial de pila de 4,13 V. c) Explicar cómo afectará al potencial de la pila una disminucion en la concentración de Co3+. Datos: Eº (Mg2+/Mg) = -2,36 V, Eº (Co3+/Co2+) = 1,81 V Solución: a) Consultar la teoría b) [Mg2+] = 0,63 M c) Consultar la teoría 8. Considerar la siguiente celda galvánica: - Semicelda I constituida por un electrodo de Pt sumergido en una disolución ácida que contiene Cr2O72- (0,4 M); Cr3+ (0,6 M) y H+ (10-3 M). 104 90

Capítulo 5. Problemas propuestos de Reacciones de Oxidación y Reducción

- Semicelda II constituida por un electrodo de Au metálico sumergido en una disolución de Au+ (0,003 M). a) Escribir y ajustar la reacción redox que tiene lugar espontáneamente, indicando cuál es el polo positivo y el polo negativo de la celda. b) Calcular la fuerza electromotriz o diferencia de potencial de la pila. c) Calcular la constante de equilibrio de la reacción redox que se produce. d) Si aumentamos la concentración de Au+ en la semicelda II, ¿cómo se modificará el potencial de la pila? Datos: Eº (Cr2O72- / Cr3+) = 1,33 V; Eº (Au+ / Au) = 1,68 V Solución: a) Semicelda I: Ánodo (-). Semicelda II: Cátodo (+) Reacción global: 2 Cr 3+ + 7 H2O + 6 Au +  Cr2O7 2- + 14 H+ + 6 Au b) E = 0,61 V c) K = 3,9·1035 d) El potencial de la pila aumentará 9. Se tiene una celda formada por dos semiceldas I y II que se unen por un puente salino y se conectan mediante un cable eléctrico. En ambos casos el electrodo es de platino. I: [ión permanganato] = 0,4 M; [protones] = 0,1 M; [ión manganeso (II)] = 0,2 M. II: [ión cloruro] = 1 M; cloro (g) a la presión de 1 atmósfera. a) Escribir y ajustar la reacción global que tiene lugar de forma espontánea. b) Calcular el potencial de la celda. c) Indicar si el potencial de la celda aumentará o disminuirá si la concentración de protones de la semicelda I se incrementa. Justificar la respuesta. Datos, a 25 ºC: Eº (ión permanganato / ión manganeso (II)) = 1,50 V Eº (cloro / ión cloruro) = 1,36 V Solución: a) 2 MnO4- + 16 H+ + 10 Cl-  2 Mn 2+ + 8 H2O + 5 Cl2 b) E = 0,05 V 105 91

Química para Ingeniería. Problemas y prácticas 

c) El potencial de la pila aumentará 10. Se diseña, a 25 ºC, una celda galvánica formada por un electrodo de platino sumergido en una disolución que contiene iones hierro (II) 0,85 M e iones hierro (III) 0,2 M y otro electrodo de platino sumergido en una disolución preparada disolviendo 1,27 g/l de yodo y 16,6 g/l de yoduro de potasio. Ambas semiceldas se conectan con un puente salino de cloruro potásico. a) Escribir la reacción que tiene lugar de forma espontánea en dicha pila e indicar la polaridad de las semiceldas ánodo y cátodo. b) Dibujar el esquema de la pila e indicar el sentido en que circularán los electrones. c) Explicar la función del puente salino. d) Determinar el potencial de la celda. e) Calcular la constante de equilibrio del proceso redox. Datos: Ar (yodo) = 127; Ar (potasio) = 39 Eº (ión hierro (III) / ión hierro (II)) = 0,77 V; Eº (yodo / ión yoduro) = 0,55 V Solución: a) Reacción global: 2 Fe 3+ + 2 I -  2 Fe 2+ + I2 Semicelda Fe 3+ / Fe 2+: Cátodo (positivo) Semicelda I2 / I - : Ánodo (negativo) b) Consultar la teoría c) Consultar la teoría c) E = 0,19 V e) K = 2,9·107 11. Una celda galvánica está formada por una semicelda I donde una tira de Pt está sumergida en una disolución de hierro (III) 0,33 M y hierro (II) 0,27 M y por otra semicelda II en la que una tira de Pt está sumergida en una disolución que contiene protones (0,01 M); permanganato potásico (0,05 M) y manganeso (II) (0,10 M). a) Escribir las semirreacciones de oxidación, de reducción y la reacción global.

106 92

Capítulo 5. Problemas propuestos de Reacciones de Oxidación y Reducción

b) Calcular el potencial de la pila. c) Dibujar un boceto del dispositivo completo de la pila indicando qué semicelda actúa como ánodo y como cátodo, cuál es el polo positivo y el polo negativo de la pila y el sentido de movimiento de los electrones. d) Calcular la constante de equilibrio del proceso redox. Datos: Eº (Fe3+ / Fe2+) = 0,77 V; Eº (MnO4- / Mn2+) = 1,49 V Solución: a) Semirreacción Oxidación: Fe 2+  Fe 3+ + 1 eSemirreacción Reducción: MnO4- + 8 H+ + 5 e-  Mn 2+ + 4 H2O Reacción global: MnO4- + 8 H+ + 5 Fe 2+  Mn 2+ + 4 H2O + 5 Fe 3+ b) E = 0,52 V c) Consultar teoría d) K = 1,0·1061 12. Para la celda galvánica formada por los siguientes electrodos: - Ánodo: Electrodo de Cu sumergido en una disolución de Cu2+. - Cátodo: Electrodo de Cl2 gas a presión de 1 atmósfera burbujeando sobre una disolución 0,1 M de iones Cl-. a) Calcular la concentración de iones Cu2+ necesaria para obtener una fuerza electromotriz de 1,122 V a 25 ºC. b) Explicar qué efecto provocará en el valor del potencial de la pila un aumento de la presión del Cl2. c) Dibujar un boceto del dispositivo completo de la pila. d) Calcular la constante de equilibrio del proceso redox. Datos: Eº (Cu2+ / Cu) = 0,34 V; Eº (Cl2 / Cl- ) = 1,358 V Solución: a) [Cu 2+ ] = 0,03 M b) El potencial de la pila aumentará c) Consultar la teoría d) K = 3,2·1034

107 93

Química para Ingeniería. Problemas y prácticas 

13. Calcular la cantidad de latas de aluminio que produce al día una fábrica, sabiendo que la celda electrolítica que le proporciona aluminio a partir de Al2O3 trabaja con una intensidad de corriente de 5000 A y que cada lata contiene 2,4 g de aluminio. Datos: Ar (Al)= 27; F = 96500 C/mol. Solución: 16787 latas de aluminio 14. Una celda electrolítica está formada por dos compartimentos conectados por un puente salino: - Un compartimento contiene agua en la que se sumerge un electrodo de platino, que actúa como ánodo. - El otro compartimento contiene una disolución de nitrato de plata en la que se sumerge un electrodo de platino, que actúa como cátodo. Ambos electrodos se conectan a una batería y se lleva a cabo la electrólisis durante 3 horas. Durante la electrólisis se observa el desprendimiento de un gas en el ánodo. Finalizada la misma, el cátodo ha experimentado un aumento de masa de 0,9 gramos. a) Escribir las reacciones que ocurren en los electrodos y la reacción global. b) Calcular la intensidad de corriente que pasa por la celda electrolítica. c) Indicar qué gas se desprende en el ánodo y calcular qué volumen ocupará a 0 ºC y 1 atmósfera de presión. Datos: Ar (Ag) = 107,9; 1 Faraday = 96500 C/mol Solución: a) Cátodo: Ag + + 1 e-  Ag Ánodo: 2 H2O  O2 + 4 H+ + 4 eReacción global: 4 Ag + + 2 H2O  4 Ag + O2 + 4 H+ b) I = 0,075 A 15. Una celda electrolítica contiene una disolución acuosa de una sal de níquel (II). Transcurrido un cierto tiempo se observa un aumento de 20,0 g en uno de los electrodos. También se observa que en el otro electrodo se desprende un gas.

108 94

Capítulo 5. Problemas propuestos de Reacciones de Oxidación y Reducción

a) Calcular la cantidad de corriente que ha pasado por la celda. b) Indicar qué gas se desprende durante el proceso y calcular el volumen desprendido del mismo a 25 ºC y 1 atmósfera de presión. Datos: Ar (Ni) = 58,7 Solución: a) Carga = 65758,09 C b) Oxígeno; V = 4,16 l de oxígeno 16. En una celda electrolítica en la que se produce la electrólisis del agua se desprenden 100 ml de hidrógeno medidos a 25 ºC y 1 atmósfera de presión. a) Escribir las semirreacciones que tienen lugar en cada electrodo indicando el ánodo y el cátodo de la celda. b) Escribir la reacción global. c) Si la celda ha trabajado con una intensidad de corriente de 1 amperio, calcular el tiempo que ha estado funcionando. d) Calcular la cantidad de agua consumida durante el proceso. Datos: Ar (O) = 16; Ar (H) = 1 Solución: a) Cátodo: 2 H+ + 2 e-  H2 Ánodo: 2 H2O  O2 + 4 H+ + 4 eb) 2 H2O  O2 + 2 H2 c) t = 789,8 segundos = 13,2 minutos d) 74 mg de H2O 17. Una fábrica obtiene sodio metálico a partir de la electrólisis del cloruro sódico fundido (celda Downs). Considerando que transcurrida 1 h se han desprendido 2600 litros de un gas medidos a 1 atmósfera de presión y 25 ºC: a) Escribir las semirreacciones que tienen lugar en cada electrodo indicando el ánodo y el cátodo de la celda. b) Escribir la reacción global. c) Calcular la cantidad de sodio obtenida y la cantidad de cloruro sódico consumida. 109 95

Química para Ingeniería. Problemas y prácticas 

d) Calcular la intensidad de corriente necesaria para obtener dicha cantidad de sodio. Datos: Ar (Na) = 23,0; Ar (Cl) = 35,5 Solución: a) Cátodo: Na+ + 1e-  Na b) Ánodo: 2 Cl-  Cl2 + 2 ec) 2 NaCl  2 Na + Cl2 d) 4894,4 g de Na; 12448,8 g de NaCl e) I = 5694,4 A 18. Los baños electrolíticos se utilizan para recubrir monedas. Algunas monedas de curso legal se hacen recubriendo con cobre un núcleo central de zinc. Si se desea bañar dichas monedas de zinc y se dispone de cobre metálico y una disolución de sulfato de cobre (II): a) Dibujar con detalle la celda que se debe utilizar y disponer los elementos en la misma para realizar el recubrimiento deseado. b) Si la superficie total de una moneda es de 7 cm2 y el espesor que se desea obtener es de 0,2 mm, calcular el tiempo que se tardará en bañar cada moneda si la intensidad de corriente que se utiliza en la celda electrolítica es de 25 A. Datos: d (cobre) = 8,9 g/cm3; Ar (cobre) = 63,5 Solución: a) Consultar la teoría b) t = 151 s

110 96



    

97



Problemas resueltos 1. Utilizar las diferentes fórmulas estructurales para representar los siguientes compuestos orgánicos: a) butano H H H H H C C C C H H H H H

CH3 CH2 CH2 CH3

CH3CH2CH2CH3

b) 2-penteno H H H H H H C C C C C H H H H

CH3 CH CH CH2 CH3

CH3CH=CHCH2CH3

c) 2-etilciclohexanol

H H C H C H

OH C H H C H

H

H H

C

C C H

H H C H H

OH

OH CH2 CH2

CH

CH2

CH CH2 CH3 CH2

OH CH2CH3

98

113

Química para Ingeniería. Problemas y prácticas



d) 3-metilbutanal H H H C H

H C H C H

H O C C H H

CH3

O

CH3 CH CH2 C H O

CH3CH(CH3)CH2CHO

H

e) propanoato de metilo H H O H H C C C O C H H H H CH3CH2COOCH3

CH3 CH2

O C O CH3 O

O

2. Justificar si son o no isómeros los siguientes pares de compuestos: a) pentano y 3-metilbutano

Sí, puesto que ambos presentan la misma fórmula molecular (C5H12). Son isómeros estructurales de cadena. b) metilciclobutano y ciclopentano

114

99

Capítulo 6. Introducción a la Química Orgánica



Sí, ya que poseen el mismo número de átomos de carbono e hidrógeno (C5H10). Son isómeros estructurales de cadena. c) 2-bromobutano y 2-bromobuteno Br

Br

No son isómeros, puesto que poseen distinto número de átomos de hidrógeno. Presentan fórmulas moleculares diferentes. d) acetato de propilo y propanoato de etilo CH 3 CO OCH 2 CH 2 C H 3

CH 3 CH 2 C O OCH 2 C H 3

Sí, son isómeros estructurales por poseer la misma fórmula molecular (C5H10O2) e) etanol y propanol CH 3 CH 2 O H

CH 3 CH 2 CH 2 OH

No son isómeros ya que presentan distinto número de átomos de carbono e hidrógeno (fórmulas moleculares diferentes)

3. Dibujar la estructura y nombrar todos los isómeros estructurales posibles de fórmula C5H11Br. Existen 8 compuestos con la misma fórmula molecular pero diferentes fórmulas estructurales.

100

115

Química para Ingeniería. Problemas y prácticas



Br

Br

Br

1-bromopentano

2-bromopentano

3-bromopentano Br

Br Br

1-bromo-2-metilbutano

2-bromo-2-metilbutano

Br

1-bromo-2,2-dimetilpropano

116

2-bromo-3-metilbutano

Br

1-bromo-3-metilbutano

101

Capítulo 6. Introducción a la Química Orgánica



Problemas para resolver 1. Utilizar las diferentes fórmulas estructurales para representar los siguientes compuestos orgánicos: a) hexano b) 2-hexeno c) 2-etilciclopentanol d) 3-metilheptanal e) acetato de etilo 2. Explicar si son isómeros o no los siguientes pares de compuestos: a)

CH3 CH2 CH2CH3

y

CH3 CHCH3 CH3

b)

y Cl Cl

c)

d)

CH3 CH2 OH

e)

CH3CO2CH2CH3

y

y

CH3CH2OCH3

y

CH3CH2CO2CH3

3. Dibujar la estructura y nombrar todos los isómeros estructurales posibles de fórmula C6H14O.

102

117

Química para Ingeniería. Problemas y prácticas



4. Formular o nombrar los siguientes compuestos: a) 2-metilbutano b) 3-hexeno c) etino d) etilbenceno e) butanoato de metilo f) propanol g) etanal h) 2-hexanona i) ácido propanoico j) CH3CH(CH3)CH2CH2CH3 k) CH3CH=CHCH(CH2CH3)CH3 l) CH2=CHCH(Cl)C(Cl)=CHCH3 m) CH3-CONH-CH3

118

103



   

104



Problemas resueltos 1. Indicar los productos que se obtienen al tratar los siguientes compuestos con Br2, luz y calor:

2. Indicar los alquenos que se forman en la deshidrobromación de los siguientes bromuros de alquilo: a) 1-bromopentano

b) 3-bromohexano

c) 2-bromo-2-metilhexano H3C

Br CH3 CH3

CH3CH2O- Na+

CH3

H C CH3 + 2 CH3 mayoritario

CH3 CH3 minoritario

121 105

Química para Ingeniería. Problemas y prácticas 

3. Predecir los productos de deshidratación de los siguientes alcoholes en presencia de un ácido y calor: a) ciclohexanol

b) 2-metil-2-pentanol

c) 2-butanol

4. Predecir los productos de las siguientes reacciones de adición: a) 2-metil-1-buteno en presencia de HCl

Sigue la regla de Markovnikov b) 2-metil-1-buteno en presencia de HBr y peróxidos

Adición anti-Markovnikov 122 106

Capítulo 7. Hidrocarburos Saturados e Insaturados 

c) 2-metil-1-buteno en presencia de H2O y H2SO4

Sigue la regla de Markovnikov

5. Escribir los productos de reacción del ciclohexeno con los siguientes reactivos: a) Br2

b) HBr

c) HBr en presencia de peróxidos

d) H2 en presencia de Pt como catalizador

123 107

Química para Ingeniería. Problemas y prácticas 

e) Br2 en presencia de luz y calor

f) Ácido peroxiacético

g) H2O / H2SO4

6. Indicar cómo llevaría a cabo las siguientes transformaciones: a) etano en etino

b) 1-clorobutano en 3-cloro-2-butanol

124 108

Capítulo 7. Hidrocarburos Saturados e Insaturados 

c) ciclohexano en 3-bromo-1,2-epoxiciclohexano

7. Predecir los productos de reacción del 1-butino con los reactivos siguientes: a) 1 mol de HBr

b) 2 moles de HCl

c) 1 mol de H2 en presencia de Pt

d) 2 moles de H2 en presencia de Pt

e) 1 mol de Cl2

125 109

Química para Ingeniería. Problemas y prácticas 

f) 2 moles de Cl2

8. Explicar cómo se podría convertir el 1-pentino en: a) 1,2-dibromo-1-penteno

b) 1,1,2,2-tetrabromopentano

c) 2-cloro-1-penteno

d) 2,2-dicloropentano

9. Indicar el producto que se obtendría al tratar acetileno con amiduro sódico seguido de los siguientes haluros de alquilo: a) 1-clorometano

126 110

Capítulo 7. Hidrocarburos Saturados e Insaturados 

b) 1-bromopropano

10. Diseñar una ruta sintética que permita transformar etano en etino.

127 111

Química para Ingeniería. Problemas y prácticas 

Problemas para resolver 1. Indicar los productos que se obtienen al tratar los siguientes compuestos con Cl2, en presencia de luz y calor:

a) b)

c)

2. Indicar los alquenos que se forman en la deshidrobromación de los siguientes bromuros de alquilo: a) 2-bromopentano b) 3-bromo-2-metilhexano 3. Dibujar la estructura de los compuestos que se obtienen en las siguientes reacciones: Cl CH3CH2O- Na+ a) C CH3

CH3 Cl

b)

C

CH2CH3

CH3CH2O- Na+

CH3 Cl

c)

CH3

C

CH2CH3

CH3CH2O- Na+

H

d) 128 112

CH3CH2CH2CH2 Cl

CH3CH2O- Na+

Capítulo 7. Hidrocarburos Saturados e Insaturados 

4. Predecir los productos de deshidratación de los siguientes alcoholes en presencia de un ácido e indicar cuál es el mayoritario: a) 1-metilciclohexanol b) 2-metilciclopentanol c) 2,2-dimetil-1-butanol 5. Predecir los productos de las siguientes reacciones de adición: a) 1-metilciclohexeno en presencia de HCl b) 1-metilciclohexeno en presencia de HBr y peróxidos c) 1-metilciclohexeno en presencia de H2O y H2SO4 6. Indicar los productos que se obtendrán al reaccionar el 1-etilciclohexeno con: a) bromo en presencia de luz y calor b) bromo a temperatura ambiente 7. Indicar los productos de reacción del 2-metil-1-buteno con los siguientes reactivos: a) HCl b) CH3COOOH c) H2 / Pd d) Br2 / h / calor e) Cl2 f) H2O / H2SO4 g) HBr / peróxido

129 113

Química para Ingeniería. Problemas y prácticas 

8. Indicar los reactivos necesarios para obtener cada una de las siguientes transformaciones:

a)

Cl

b)

Cl Cl

c)

OH

9. Indicar el producto que se obtendrá al tratar propino con amiduro sódico seguido de los siguientes haluros de alquilo: a) 1-cloropropano b) 1-bromobutano 10. Indicar cómo obtener 1,2-diciclohexileteno a partir de acetileno y ciclohexano. 11. Dibujar la estructura de los productos que se obtienen en cada una de las siguientes reacciones:

130 114

Capítulo 7. Hidrocarburos Saturados e Insaturados 

12. Indicar los productos que se forman a partir del 1-buteno en las siguientes reacciones consecutivas: i) Br2; ii) 2 moles de amiduro sódico; iii) amiduro sódico, iv) yoduro de metilo; v) un mol de hidrógeno empleando Pd como catalizador; vi) perácido. 13. Indicar cómo se llevarán a cabo las siguientes transformaciones: a) 1-pentanol en 2-bromopentano b) 1-clorobutano en 3-cloro-2-butanol c) acetileno en 2-buteno

131 115



  

116



Problemas resueltos 1. Indicar los productos que se obtienen en las siguientes reacciones: a) benceno con Cl2 y AlCl3 Cl2 / AlCl3

Cl

b) benceno con HNO3 y H2SO4

HNO3 / H2SO4

NO2

c) benceno con SO3 en presencia de H2SO4

SO3 / H2SO4

SO3H

d) benceno con clorometano y AlCl3

CH3Cl / AlCl3

CH3

135 117

Química para Ingeniería. Problemas y prácticas 

e) benceno con cloruro de benzoilo y AlCl3 O C

PhCOCl / AlCl3

2. Indicar los productos que se obtienen al tratar con HNO3 / H2SO4 los siguientes compuestos: a) hidroxibenceno (fenol) OH

OH

OH NO2

HNO3 / H2SO4

+ NO2

(mayoritario) El sustituyente –OH es / dirigente. b) bromobenceno Br

Br HNO3 / H2SO4

Br NO2 + NO2

(mayoritario) El sustituyente –Br es / dirigente.

136 118

Capítulo 8. Hidrocarburos Aromáticos

c) ácido benzoico

COOH

COOH HNO3 / H2SO4

NO2 El sustituyente –COOH es  dirigente. d) metilbenceno (tolueno) CH3

CH3 HNO3 / H2 SO4

CH3 NO2 + NO2

(mayoritario) El sustituyente –CH3 es / dirigente. e) ácido bencenosulfónico SO3H

SO3H HNO3 / H2SO4

NO2

El sustituyente –SO3H es  dirigente.

137 119

Química para Ingeniería. Problemas y prácticas 

3. Indicar los productos que se obtienen en las siguientes reacciones: a) bromobenceno con amiduro sódico Br Na NH2

No hay reacción

La reacción de los hidrocarburos aromáticos es la sustitución electrofílica, por tanto el bromobenceno no reacciona con NH2� por ser un nucleófilo. b) bromobenceno con bromuro de isopropilo y AlCl3 Br

Br

Br

CH3 CH

(CH3 )2CHBr / AlCl3

CH3

+ H3C CH CH3

(mayoritario) El sustituyente –Br es / dirigente. c) bromobenceno con bromo y AlCl3 Br

Br

Br Br2 / AlCl3

Br + Br

(mayoritario) El sustituyente –Br es / dirigente. 138 120

Capítulo 8. Hidrocarburos Aromáticos

d) bromobenceno con propinuro sódico Br CH3 C

C

Na

No hay reacción

Por la misma razón que en el apartado (a), los hidrocarburos aromáticos no reaccionan con alquinuros (CH3-C�C � ) por ser nucleofilos.

4. Indicar el producto o productos que se forman en las siguientes reacciones: a) 3-fenilciclohexeno con bromo, luz y calor B r2

+

h / 

Br

Br

(mayoritario) Las condiciones de reacción son de halogenación de alcanos. El primero de los productos será el mayoritario por estar la posición de halogenación doblemente favorecida, ya que se trata de una posición bencílica y alílica. b) 3-fenilciclohexeno con bromo Br

Br

Br2

Se trata de una reacción de adición electrofílica (halogenación) de alquenos.

139 121

Química para Ingeniería. Problemas y prácticas 

c) 3-fenilciclohexeno con bromo y tricloruro de aluminio Br2 / AlCl3

Br

+ Br

(mayoritario) La presencia del AlCl3 proporciona las condiciones para que se produzca la reacción de halogenación del benceno monosustituído. El sustituyente (ciclohexeno) es / dirigente. El producto halogenado en posición  será el mayoritario. d) 3-fenilciclohexeno con bromuro de hidrógeno Br HBr

Br +

Reacción de hidrohalogenación de alquenos. Se obtiene una mezcla de dos productos halogenados.

5. Predecir los productos que se obtienen a partir del tratamiento de los siguientes compuestos con permanganato de potasio y calor: a) -butilbenceno

KMnO4 

COOH

Las condiciones de reacción son de oxidación bencílica de la cadena lateral del anillo bencénico.

140 122

Capítulo 8. Hidrocarburos Aromáticos

b) 2-fenilpropeno KMnO4 

No hay reacción

Para que se produzca la oxidación en la cadena lateral del anillo bencénico es necesario que haya al menos un hidrógeno en la posición bencílica. c) 1,2-dimetilbenceno (-xileno) COOH KMnO4

COOH



Con estas condiciones de reacción se produce la oxidación de toda cadena lateral del anillo bencénico que posea hidrógenos en posición bencílica.

6. Indicar qué productos se forman en las siguientes reacciones: a) metilbenceno en presencia de cloro, luz y calor CH3

CH2 Cl Cl2 h

Las condiciones de reacción son de halogenación bencílica. Se trata de una reacción de la cadena lateral del anillo bencénico.

141 123

Química para Ingeniería. Problemas y prácticas 

b) metilbenceno con ácido sulfúrico fumante CH3

CH3

CH3 SO 3H

SO 3 / H2SO4

+ SO3H

(mayoritario) Las condiciones de reacción son de sulfonación del anillo bencénico. Se trata de una reacción de sustitución aromática electrofílica en el anillo de benceno que está monosustituído. El sustituyente –CH3 es / dirigente. c) metilbenceno con cloruro de acetilo y tricloruro de aluminio CH3

CH3

CH3

O C

CH3COCl / AlCl3

CH3

+ C

O

CH3

(mayoritario) Las condiciones de reacción son de acilación de Friedel-Crafts del anillo de benceno. Al igual que en el apartado (b), se trata de una reacción de sustitución aromática electrofílica sobre un benceno monosustituído con un sustituyente / dirigente.

142 124

Capítulo 8. Hidrocarburos Aromáticos

d) metilbenceno con permanganato potásico y calor CH3

COOH KMnO4 

Las condiciones de reacción son de oxidación bencílica. Se trata de una reacción de la cadena lateral del anillo de benceno.

7. A partir de benceno y los reactivos necesarios sintetizar 3-acetiltolueno. COCH3

COCH3 CH3COCl / AlCl3

CH3Cl / AlCl3 CH3

8. Obtener a partir de benceno o tolueno y los reactivos necesarios los siguientes compuestos: a) ácido 3-clorobencenosulfónico SO3H SO3 / H2 SO4

SO3H Cl2 / AlCl3 Cl

b) fenil metil cetona O CH3COCl / AlCl3

C

CH3

143 125

Química para Ingeniería. Problemas y prácticas 

c) ácido 4-nitrobenzoico CH3

CH3

COOH

HNO 3 / H2SO 4

KMnO4  NO2

NO 2

La nitración del tolueno conducirá a la mezcla de los productos  y , pero se continúa con el  por ser el mayoritario.

9. Sintetizar los siguientes productos a partir de benceno y los reactivos necesarios: a) vinilbenceno (o etenilbenceno)

Cl

CH2CH3

CH

CHCH3

CH3CH2 Cl / AlCl3

Cl2

CH3CH2O Na

h



CH2

b) -acetiletilbenceno O C CH3COCl / AlCl3

O CH3

C

CH3

CH3CH2Cl / AlCl3 CH2CH3

144 126

Capítulo 8. Hidrocarburos Aromáticos

c) -acetiletilbenceno CH2CH3 CH3CH2Cl / AlCl3

CH2 CH3

CH2CH3 C

CH3COCl / AlCl3

CH3

O C

CH3

O (mayoritario)

10. Indicar el producto o productos que se obtienen en las siguientes reacciones: a) -viniltolueno con bromo, luz y calor CH2 Br

CH3 Br2 CH

CH2

h

CH CH2

b) -viniltolueno con perácido CH3

CH3 RCOOOH O CH

CH2

CH CH2

c) -viniltolueno con bromuro de hidrógeno CH3

CH3 HBr CH

CH2

CH

CH3

Br 145 127

Química para Ingeniería. Problemas y prácticas 

d) -viniltolueno con bromo CH3

CH3 Br2 CH

CH2

CH CH2 Br Br

e) -viniltolueno con agua y ácido sulfúrico CH3

CH3 H2O / H2SO4 CH

CH2

CH CH3 OH

f) -viniltolueno con bromuro de hidrógeno y peróxidos CH3

CH3 HBr / ROOR' CH

CH2

CH2 CH2 Br

11. Sintetizar 3-fenil-2-propanol a partir de cloruro de bencilo y acetiluro sódico.

CH2 Cl

1 mol H2 / Pt

146 128

HC

C Na+

CH2 CH

CH2

CH2 C

H2O / H2SO4

CH

OH CH2 CH

CH3

Capítulo 8. Hidrocarburos Aromáticos

Problemas para resolver 1. Indicar los productos que se obtienen al tratar el benceno con los siguientes reactivos: a) Br2 / AlCl3 b) HNO3 / H2SO4 c) SO3 / H2SO4 d) yoduro de etilo / AlCl3 e) cloruro de acetilo / AlCl3  2. Predecir los productos de las siguientes reacciones: a) clorociclohexano en presencia de benceno y AlCl3 b) 2-cloropropano en presencia de benceno y AlCl3 3. Indicar los productos que se obtienen al tratar con HNO3 / H2SO4 los siguientes compuestos: a) metoxibenceno b) yodobenceno c) benzaldehído d) nitrobenceno 4. Indicar los productos que se obtienen al tratar bromobenceno con: a) Na+ OHb) Br2 / AlCl3 c) SO3 / H2SO4 d) HC�C- Na+

147 129

Química para Ingeniería. Problemas y prácticas 

5. Indicar los productos de la reacción del 1-fenil-2-buteno en cada una de las siguientes condiciones: a) Br2 en presencia de luz y calor b) Br2 c) Br2 / AlCl3 6. Predecir los productos que se obtienen a partir del tratamiento de los siguientes compuestos con permanganato de potasio y calor: a) isopropilbenceno b) 1,4-dimetilbenceno (-xileno) c) -butilbenceno 7. Indicar los productos que se obtienen al tratar el vinilbenceno con cada uno de los siguientes reactivos: a) Br2 en presencia de luz y calor b) CH3CO3H c) Cl2 / AlCl3 d) Br2 e) SO3 / H2SO4 f) cloruro de acetilo / AlCl3 g) HBr / ROOR’ h) permanganato potásico y calor 8. Proponer una ruta de síntesis para obtener 3-acetilclorobenceno a partir de benceno y los reactivos necesarios. 9. Indicar la mejor ruta sintética para obtener los siguientes compuestos: a) Etilbenceno a partir de benceno, etileno y los reactivos necesarios. b) Vinilbenceno (o etenilbenceno) a partir de etilbenceno y los reactivos necesarios.

148 130

Capítulo 8. Hidrocarburos Aromáticos

10. Obtener a partir de benceno o tolueno los siguientes compuestos: a) 4-clorometilnitrobenceno b) ácido 3-nitrobenzoico c) ácido 4-bromobenzoico 11. A partir de tolueno y 2-buteno obtener ácido 3-(1-metilpropil)benzoico. 12. Sintetizar 1,4-difenil-2-butino a partir de tolueno y acetileno.

149 131



&DStWXOR3UREOHPDVSURSXHVWRVGH+LGURFDUEXURV6D WXUDGRV,QVDWXUDGRV\$URPiWLFRV

132



1. Indicar los productos que se forman cuando el etano reacciona consecutivamente con los siguientes reactivos: (i) cloro, luz y temperatura; (ii) etóxido sódico en caliente; (iii) bromo; (iv) dos moles de amiduro sódico 2. Dibujar la estructura de los productos que se obtienen en cada una de las siguientes reacciones:

3. Proponer una ruta sintética para los siguientes productos utilizando compuestos de 2 carbonos y los reactivos necesarios: a) etano en ácido etanoico b) tolueno en ácido 3-vinilbenzoico 4. Indicar los productos que se obtienen al tratar consecutivamente 2-fenil-2propanol con cada uno de los siguientes reactivos: a) ácido sulfúrico y calor, b) cloruro de metilo y AlCl3, c) permanganato potásico y calor, d) bromuro de hidrógeno en presencia de peróxidos, e) acetiluro sódico

153 133

Química para Ingeniería. Problemas y prácticas 

5. Completar el siguiente esquema de reacciones:

6. Indicar los productos que se forman al tratar el benceno con los siguientes reactivos de forma consecutiva: a) yoduro de etilo y AlCl3, b) cloruro de acetilo y AlCl3, c) cloro, luz y temperatura, d) etóxido sódico y calor, e) ácido bromhídrico en presencia de peróxidos 7. Empleando los reactivos necesarios, indicar cómo se puede convertir: a) propano en propino b) tolueno en ácido 3-etilbenzoico 8. a) Indicar una ruta sintética para transformar vinilbenceno en fenilacetileno. b) Indicar los productos de reacción del 3-fenil-1-metilciclohexeno con cada uno de los siguientes reactivos: b1) cloro b2) cloro, luz y calor b3) cloro en presencia de tricloruro de aluminio

154 134

Capítulo 9. Problemas propuestos de Hidrocarburos Saturados, Insaturados y Aromáticos

b4) bromuro de hidrógeno b5) bromuro de hidrógeno en presencia de peróxidos 9. a) Proponer una síntesis para obtener el 1-cloro-3-propanoilbenceno a partir de benceno. b) Indicar los productos que se obtienen al tratar consecutivamente 1-butino con: i) Na+ NH2- ; ii) BrCH2CH3; iii) 1 mol H2 / Pt; iv) Cl2 / h / . 10. a) Sintetizar -acetiletilbenceno a partir de benceno y los reactivos necesarios. b) Indicar los productos que se obtendrán cuando los siguientes compuestos reaccionen con etóxido sódico en caliente: b1) 1-cloropropano b2) 2-cloro-2-(4-clorofenil)propano b3) 2-metil-bromociclohexano b4) bromuro de -hidroxibencilo 11. a) A partir de tolueno y 2-metilpropeno obtener el ácido 4-(1metiletil)benzoico. b) Indicar los productos que se obtendrán en las siguientes reacciones:

155 135

Química para Ingeniería. Problemas y prácticas 

12. A partir de benceno y los reactivos necesarios obtener: a) ácido 4-nitrobenzoico b) ácido 3-acetilbenzoico c) ácido -etilbencenosulfónico 13. A partir de benceno, compuestos de 1 ó 2 carbonos y los reactivos necesarios proponer una síntesis para obtener: a) ácido 3-isopropilbenzoico b) 4-metilbromobenceno 14. a) Indicar los productos que se forman en la siguiente secuencia sintética:

b) A partir de ciclohexano obtener 1,3-ciclohexadieno. 15. Indicar cómo obtener los siguientes productos: a) 2-bromobutano a partir de acetileno y los reactivos necesarios b) 2-bromopentano a partir de 1-pentanol y los reactivos necesarios

156 136

Capítulo 9. Problemas propuestos de Hidrocarburos Saturados, Insaturados y Aromáticos

16. a) Indicar los productos que se obtienen en cada paso de la siguiente síntesis:

b) Sintetizar los siguientes compuestos a partir de benceno y los reactivos necesarios: b1) ácido 3-clorobencenosulfónico b2) 1-bromo-4-acetilbenceno 17. a) Indicar el producto o productos que resultan al reaccionar etóxido sódico en caliente con: a1) 2-cloro-2-fenilbutano a2) bromuro de ciclohexilo b) A partir de benceno y de los reactivos necesarios obtener ácido 2clorobenzoico. 18. a) Indicar los productos que se forman al tratar con etóxido sódico en caliente los siguientes compuestos: a1) clorociclohexano a2) 1-clorometilbenceno a3) 2-bromo-2-fenilpropano a4) 2-fenil-1-cloropropano b) A partir de benceno y de los reactivos necesarios obtener: b1) 1,4-dibromobenceno b2) ácido 4-acetilbenzoico

157 137



     

138



Problemas resueltos 1. Ordenar por punto de ebullición decreciente los siguientes compuestos: metanol; pentanol; dietil éter; 1,2-etanodiol; 1,2,3-propanotriol. El punto de ebullición es una propiedad física de los compuestos que depende de las fuerzas intermoleculares. Los compuestos con mayores fuerzas intermoleculares tendrán mayores puntos de ebullición, ya que se requerirá mayor energía para vencer la atracción entre las moléculas del líquido y convertirlo en vapor. La interacción intermolecular entre los alcoholes es del tipo puente de hidrógeno. Al aumentar el número de grupos OH aumenta el número de interacciones por puente de hidrógeno. Así el 1,2,3-propanotriol con tres grupos OH, es el que presenta el punto de ebullición más alto seguido del 1,2-etanodiol. El punto de ebullición de los alcoholes con el mismo número de OH aumenta con el tamaño de la molécula, por lo que el pentanol con cinco carbonos, tiene un punto de ebullición más alto que el metanol que sólo tiene un carbono. Finalmente, el compuesto de menor punto de ebullición es el dietil éter puesto que las interacciones intermoleculares en los éteres son del tipo dipolo-dipolo con una fuerza de enlace mucho menor que la de puente de hidrógeno. OH OH

OH >

OH OH CH2 -CH2 > CH 3CH2 CH2 CH 2CH 2OH > CH3 OH > CH CH OCH CH 3 2 2 3

2. Indicar como transformar: a) 2-butanona en 2-butanol

Reacción de reducción b) propanal en propanol

Reacción de reducción 161 139

Química para Ingeniería. Problemas y prácticas 

c) 1-cloropropano en propanol

Reacción de sustitución d) bromociclopentano en ciclopentanol

Br

OH NaOH

Reacción de sustitución

3. Indicar cómo transformar: a) fenilmetanol en benzaldehído

Reacción de oxidación b) fenilmetanol en ácido benzoico

Reacción de oxidación

162 140

Capítulo 10. Alcoholes, Ésteres y Epóxidos 

c) ciclohexanol en ciclohexanona

Reacción de oxidación

4. Indicar como transformar: a) butanol en bromobutano

Reacción de sustitución b) benzaldehído en cloruro de bencilo

Reacción de reducción y posterior sustitución c) cloruro de bencilo en benzaldehído

Reacción de sustitución y posterior oxidación

163 141

Química para Ingeniería. Problemas y prácticas 

5. Indicar los productos que se obtienen al hacer reaccionar: a) cloropropano con etóxido sódico

Los haluros primarios y secundarios en presencia de alcóxidos forman éteres: reacción de Williamson. b) bromuro de etilo con hidróxidosódico

Los haluros primarios y secundarios reaccionan con hidróxidos alcalinos dando alcoholes. c) cloruro de -butilo con etóxido sódico y calor

Los haluros terciarios en presencia de bases (alcóxidos alcalinos) sólo sufren deshidrohalogenación dando lugar al alqueno correspondiente.

6. A partir de los precursores inmediatos proponer un esquema sintético para los siguientes éteres: a) etil fenil éter

164 142

Capítulo 10. Alcoholes, Ésteres y Epóxidos 

b) ciclohexilmetil etil éter

7. Proponer un esquema sintético para obtener dipropil éter a partir de propanol

8. A partir de 4-metilfenol sintetizar el 4-metoxifenilmetanol

165 143

Química para Ingeniería. Problemas y prácticas 

9. Indicar como transformar: a) propano en 2-propanol

b) 2-buteno en 2-butanona

CH3CH=CHCH3

H2O / H 2 SO 4

OH CH3 CHCH2CH3

KMnO 4

O CH3CCH 2CH 3

c) 2-buteno en butanal

d) propano en propanal CH3CH2CH3

Cl2 h

Cl CH3CH2CH2Cl + CH3CHCH3

CH3CH2O-Na+ CH3CH=CH2

CH3CH2CH2OH 166 144

HBr ROOR´

CrO3 piridina

CH3CH2CH2Br NaOH

CH3CH2CHO

Capítulo 10. Alcoholes, Ésteres y Epóxidos 

Problemas para resolver 1. Indicar como se pueden llevar a cabo las siguientes transformaciones: a) a partir de 3-metil-2-pentanona obtener 3-metil-2-pentanol b) a partir de 2,3-dimetil-2-buteno obtener 2,3-dimetil-2-butanol c) a partir de 2-metil-cloropentano obtener 2-metilpentanol 2. Dibujar la estructura de los productos que se obtienen en la deshidratación en medio ácido de los siguientes alcoholes: a) 2-metilciclohexanol b) 2-feniletanol c) fenilmetanol d) fenol 3. Indicar el reactivo (o reactivos) necesarios para transformar el ciclopentanol en: a) ciclopentanona b) clorociclopentano c) ciclopenteno d) 3-bromociclopenteno 4. Escribir el producto que se obtiene en las siguientes reacciones: a) butanol con NaOH b) butanol con Na c) fenol con NaOH seguido de la adición de cloruro de etilo 5. Indicar como transformar el ciclohexeno en: a) ciclohexanol b) ciclohexanona c) 2-ciclohexen-1-ol 167 145

Química para Ingeniería. Problemas y prácticas 

6. Sintetizar a partir de butanal y los reactivos necesarios, los siguientes compuestos: a) 1-butanol b) 1,2-butanodiol c) ácido butanóico d) dibutil éter 7. Indicar una ruta sintética para preparar a partir de 2-buteno y los reactivos necesarios el siguiente compuesto:

O Cl 8. Sintetizar a partir de 1-clorociclopentano y los reactivos necesarios 1,2dihidroxiciclopentano 9. Escribir el mejor método para la preparación de los siguientes éteres mediante la síntesis de Williamson: a) metoxibenceno b) bencil tert-butil éter c) metil vinil éter d) metil propil éter 10. Indicar el producto que se obtiene al tratar óxido de etileno con: a) cloruro de metilmagnesio b) bromuro de fenilmagnesio c) cloruro de etilo con magnesio d) cloruro de vinilmagnesio 11. Indicar como obtener pentanol a partir de óxido de etileno y propeno 168 146





148



Problemas resueltos 1. Indicar como transformar: a) etanol en etanal

Reacción de oxidación de un alcohol primario a aldehído b) 2-propanol en 2-propanona

Reacción de oxidación de un alcohol secundario a cetona c) 3-fenilpropeno en 1-fenil-2-propanona

Reacción de hidratación seguida de oxidación a cetona

2. Indicar un compuesto carbonílico y un reactivo de Grignard adecuado para sintetizar: a) 1,1-difeniletanol

171 149

Química para Ingeniería. Problemas y prácticas 

b) fenilmetanol

c) 1-etilciclopentanol

d) 2-ciclohexil-2-butanol

3. A partir de ciclohexano, compuestos de 1 ó 2 carbonos y los reactivos necesarios sintetizar el 2-(2-ciclohexenil)etanol.

172 150

Capítulo 11. Aldehidos y Cetonas 

4. Sintetizar a partir de benceno, compuestos de 1 ó 2 carbonos y los reactivos necesarios el 2-(4-etilfenil)-2-butanol.

5. A partir de etanol y los reactivos necesarios sintetizar el ácido hexanoico.

6. A partir del ácido 2-feniletanoico, compuestos de 1 ó 2 carbonos y los reactivos necesarios obtener el ácido 4-fenilbutanoico.

173 151

Química para Ingeniería. Problemas y prácticas 

7. Indicar los compuestos que se forman al tratar consecutivamente bromuro de ciclohexilo con: i) etóxido sódico /  ii) H2O / H2SO4, iii) KMnO4, iv) CH3MgBr y posteriormente tratado con agua.

8. Indicar los compuestos que se forman al tratar consecutivamente el ciclohexeno con: i) Cl2 / hii) Mg / éter, iii) butanal, iv) H2O, v) H2SO4

174 152

Capítulo 11. Aldehidos y Cetonas 

Problemas para resolver 1. Indicar el alcohol de partida y el reactivo necesario para obtener los siguientes compuestos: a) propanona b) fenil propil cetona c) benzaldehído d) ácido benzoico 2. Indicar qué productos se obtienen al tratar 3-ciclohexenona con: a) NaBH4 b) 1 mol de hidrógeno / Pd c) un perácido d) bromuro de etilmagnesio 3. Indicar los productos que se obtienen al tratar benzaldehído con: a) KMnO4 b) cloruro de acetilo / AlCl3 c) NaBH4 d) bromuro de fenilmagnesio y posteriormente agua 4. Indicar los productos que se forman al hacer reaccionar consecutivamente fenilmetanol con: i) CrO3 / piridina, ii) CH3CH2MgBr, iii) H2O, iv) H2SO4 / , v) perácido. 5. Indicar el producto que se obtiene al hacer reaccionar los siguientes compuestos con bromuro de metilmagnesio y posteriormente con agua: a) formaldehído b) acetaldehído c) benzaldehído d) acetona 175 153

Química para Ingeniería. Problemas y prácticas 

6. Escribir la síntesis más adecuada para obtener etil fenil cetona a partir de benceno y ácido propanoico. 7. A partir de 2-fenilacetaldehído y los reactivos necesarios obtener: a) 1-fenil-2-butanol b) 1-fenil-2-butanona 8. Indicar cómo sintetizar 1-fenil-2-metil-2-propanol a partir de tolueno y acetona. 9. Sintetizar 1-fenil-1-hexanol a partir de benceno y ácido hexanoico.

176 154



    

155



Problemas resueltos 1. Justificar por qué el diclorometano no es soluble en agua mientras que el ácido acético es completamente miscible en ella. El ácido acético es totalmente soluble en agua porque es una molécula pequeña capaz de formar enlaces por puente de hidrógeno con el agua a través de ambos oxígenos, mientras que el diclorometano, aunque es una molécula con cierta polaridad debido a los enlaces C-Cl, no puede formar puente de hidrógeno con el agua. Considerar que los únicos átomos capaces de interaccionar a través de enlaces por puente de hidrógeno son: O, N y F.

2. Indicar como transformar: a) ciclohexilmetanol en ácido ciclohexilmetanoico CH 2OH

COOH KMnO 4

b) butanal en cloruro de butanoilo

CH3CH2CH2CHO

KMnO4

CH3CH2CH2COOH

SOCl2 CH CH CH COCl 3 2 2

c) butanol en butanoato de butilo CH 3CH2 CH2CH 2OH CH 3CH2 CH2COCl

156

KMnO4

CH 3CH 2CH2COOH

CH 3CH 2CH2CH 2OH

SOCl2

CH 3CH2CH2COCl

CH 3CH2 CH2COOCH2CH 2CH 2CH3

179

Química para Ingeniería. Problemas y prácticas



d) butanol en anhídrido butanoico

CH3CH2CH2CH2OH

CH3CH2CH2COCl

KMnO4

CH3CH2CH2COOH

CH3CH2CH2COOH

SOCl2

CH3CH2CH2COOCOCH2CH2CH3

3. Indicar el producto que se forma al hacer reaccionar: a) cloruro de propanoilo con 4-metoxifenol OH CH 3 CH 2COCl +

CH3 CH 2 COO

OCH 3

OCH3

b) cloruro de propanoilo con ácido 4-metoxibenzoico COOH CH 3CH 2COCl +

CH3CH2COOCO

OCH 3

OCH3

c) cloruro de propanoilo con 4-metoxiaminobenceno NH 2 CH3 CH 2 COCl +

CH 3 CH 2CONH

OCH3

OCH 3

d) cloruro de propanoilo con agua

CH3CH2COCl + 180

H 2O

CH3CH2COOH 157

Capítulo 12. Ácidos Carboxílicos y Derivados



4. A partir de etanol y los reactivos necesarios obtener: a) ácido butanoico O 1) Mg CH 3 CH 2OH CH 3CH 2Cl CH 3 CH 2MgCl éter 2) H2 O KMnO 4 CH3 CH2 CH 2COOH CH 3CH2 CH2 CH 2OH HCl

b) ácido propanoico

CH3CH2OH 1) HCHO 2) H2O

HCl

CH3CH2Cl

CH3CH2CH2OH

Mg CH3CH2MgCl éter

KMnO4

CH3CH2COOH

c) N,N-dimetilacetamida CH3CH2OH

KMnO4

HN CH3COCl

CH3COOH

SOCl2

CH3 CH3

CH3CON(CH3)2

d) acetato de etilo

158

181

Química para Ingeniería. Problemas y prácticas



5. Indicar el producto que se obtiene a partir del 1-pentanol en cada uno de los pasos de la siguiente serie consecutiva de reacciones: i) permanganato potásico; ii) cloruro de tionilo; iii) metilamina. CH 3CH 2CH2CH2CH 2OH

SOCl2

KMnO4 CH CH CH CH COOH 3 2 2 2

CH3 CH 2CH 2CH2COCl

CH 3-NH 2

CH 3CH2 CH2 CH 2CONHCH 3

6. A partir de tolueno, compuestos de 1 ó 2 carbonos y los reactivos necesarios sintetizar: a) anhídrido del ácido feniletanoico

182

159



Capítulo 12. Ácidos Carboxílicos y Derivados

b) feniletanoato de 2-feniletilo Para sintetizar el éster partiremos del ácido feniletanoico y del 2-feniletanol obtenidos en el apartado a)

7. A partir de ciclohexano y los reactivos necesarios sintetizar el ácido 2,3dibromociclohexilmetanoico.

8. A partir de eteno y los reactivos necesarios sintetizar la N-etilbutanamida

160

183

Química para Ingeniería. Problemas y prácticas



Problemas para resolver 1. Indicar los compuestos que se forman en las siguientes reacciones: a) pentanol con permanganato potásico b) pentanol con CrO3 / piridina c) propilbenceno con permanganato potásico a elevada temperatura d) benzaldehído con permanganato potásico 2. A partir de 2-ciclohexiletanol y los reactivos necesarios obtener: a) ácido 2-ciclohexiletanoico b) cloruro de 2-ciclohexiletanoilo c) 2-ciclohexiletanoato de etilo d) 2-ciclohexiletanamida e) anhídrido del ácido 2-ciclohexiletanoico 3. Indicar como transformar ácido fenilacético en: a) 2-feniletanol b) fenilacetato de etilo c) 2-fenilacetaldehído 4. A partir de benceno y los reactivos necesarios sintetizar: a) ácido benzoico b) benzoato sódico c) fenilmetanol d) benzoato de metilo

184

161

Capítulo 12. Ácidos Carboxílicos y Derivados



5. Indicar los reactivos necesarios para realizar las siguientes transformaciones: propanol

ácido propanoico

cloruro de propanoilo

propanoato de etilo

anhídrido propanoico

N-metil propanamida

6. A partir de etileno, butanol y los reactivos necesarios obtener acetato de butilo. 7. Obtener ácido 5-fenilpentanoico a partir de 3-fenil-1-cloropropano y óxido de etileno.

162

185



                  

163



1. Indicar como transformaría el ácido fenilacético en: a) 4-cloro-2-feniletanol b) N-metil-2-fenilacetamida c) 2-fenilacetato de metilo 2. Indicar como transformar propano en: a) 1-propeno b) 1-propanol c) propanal d) ácido propanoico 3. Indicar los compuestos que se obtienen al hacer reaccionar consecutivamente el estireno con 1) HBr/peróxidos, 2) NaOH, 3) KMnO4, 4) etilamina. 4. A partir de ciclopentano, compuestos de dos carbonos y los reactivos necesarios obtener 4-ciclopentil-1-butanol. 5. A partir de benceno, propanol y los reactivos necesarios obtener: a) ácido benzoico b) propilbenceno c) etil fenil cetona d) 1-fenil-1-propanol 6. A partir de cloruro de bencilo, compuestos de 1 ó 2 carbonos y los reactivos necesarios proponer una síntesis para el ácido 3-fenilpropanoico. 7. Proponer una ruta sintética para llevar a cabo las siguientes transformaciones: a) fenilacetileno a partir de etilbenceno b) propanona a partir de propano c) 1-butanol a partir de etileno y acetileno 164

189

Química para Ingeniería. Problemas y prácticas



8. Indicar cómo se pueden transformar los siguientes compuestos: a) cloruro de bencilo en ácido fenilacético b) ácido benzoico en cloruro de benzoilo c) propanol en propanoato de propilo 9. Sintetizar 4-fenil-2-butanol a partir de 2-fenilbromoetano y compuestos de 2 átomos de carbono. 10. Indicar los productos de reacción del bromuro de etil magnesio con cada uno de los siguientes productos: a) -butanol b) propanona c) tolueno d) óxido de etileno 11. Indicar en cada caso el compuesto carbonílico y el reactivo de Grignard adecuados para preparar los siguientes alcoholes: a) butanol b) 2-fenil-2-propanol c) 1-metilciclohexanol d) difenil metanol 12. A partir de cloruro de bencilo y compuestos de 2 carbonos obtener 3fenilpropanoato de etilo. 13. Indicar cómo obtener benzoato de etilo a partir de tolueno y etileno. 14. A partir de fenol y butanol obtener el butil fenil éter.

190

165

Capítulo 13. Problemas propuestos de Alcoholes, Éteres, Epóxidos, Aldehídos, Cetonas, Ácidos y Derivados 

15. Indicar cómo sintetizar el 1,1-difeniletanol a partir de 1-feniletanol y clorobenceno. 16. Indicar cómo realizar las siguientes transformaciones: a) etanol en acetaldehido b) etanol en cloroetano c) etanol en anhídrido acético d) etenilbenceno en fenil metil cetona 17. A partir de propanol, benceno y los reactivos necesarios obtener: a) 1-fenilpropano b) etil fenil cetona 18. A partir de ciclohexano y compuestos de 1 ó 2 carbonos proponer la síntesis de 2-ciclohexiletanol. 19. Indicar cómo realizar las siguientes transformaciones: a) etileno en ácido acético b) etanol en cloruro de acetilo c) etanol en acetamida d) acetamida en ácido acético 20. Indicar cómo realizar las siguientes transformaciones: a) fenol en metoxibenceno b) ácido benzoico en anhídrido benzoico c) metoxibenceno en ácido 4-metoxibenzoico d) propeno en 2-propen-1-ol

166

191



Parte II. Prácticas

167



Práctica 1. Termodinámica Química

169



1. Introducción El calor absorbido o desprendido en una reacción química se puede determinar experimentalmente utilizando dispositivos denominados calorímetros. Existen distintos tipos de calorímetros dependiendo del tipo de calor que se pretende medir (calor de disolución, combustión, hidrogenación, etc). Entre ellos, se encuentran el calorímetro de bomba que está especialmente diseñado para determinar el calor desprendido en una reacción de combustión y el calorímetro de taza, que nos permite determinar el calor que acompaña a las reacciones que tienen lugar en disolución acuosa.

Esquema de un calorímetro de bomba

Esquema de un calorímetro de taza El funcionamiento del calorímetro, se basa en la medida de la variación de temperatura que experimenta una determinada cantidad de agua por efecto del calor desprendido o absorbido en una reacción química.

170

197

Química para Ingeniería. Problemas y prácticas



Para determinar la cantidad de calor liberada en el proceso, tendremos en cuenta que el calor liberado en la reacción es absorbido por la disolución y por los componentes del calorímetro, por lo tanto tendremos que: –Hdesp = Habs = m·ce· (Tf – Ti) + K·(Tf – Ti)

[I]

siendo: m: masa de la disolución ce: calor especifico de la disolución Ti: temperatura inicial Tf: temperatura final K: constante del calorímetro

2. Objetivo Determinar el calor desprendido en una serie de procesos en disolución.

3. Parte experimental En esta práctica se pretende determinar: 1) El calor desprendido en la disolución del NaOH (s) en agua: �H1 NaOH (s) + H2O �

Na+ (aq) + OH- (aq)

2) El calor desprendido en la neutralización del NaOH (aq) con HCl (aq): �H2 Na+ (aq) + OH- (aq) + H+ (aq) + Cl- (aq) � H2O + Na+ (aq) + Cl- (aq) 3) El calor desprendido en el proceso de disolución del NaOH (s) y su neutralización con HCl (aq): �H3 NaOH (s) + H+ (aq) + Cl- (aq) � H2O + Na+ (aq) + Cl- (aq) Para determinar el calor liberado en cada uno de los procesos anteriores es necesario calcular el calor absorbido por el calorímetro. Para ello, en primer lugar se procede al calibrado del calorímetro, lo que permite obtener la constante del calorímetro (K).

198

171

Práctica 1. Termodinámica Química



3.1. Calibrado del calorímetro Se introducen 100 ml de agua destilada en el calorímetro y se mide su temperatura (Ti). A continuación, se miden exactamente 100 ml de agua, se introducen en un vaso de precipitados y se calientan en una placa calefactora hasta una temperatura 15 ºC superior a Ti. Alcanzada dicha temperatura, se retira el vaso de la placa calefactora y se espera unos minutos para que se estabilice la temperatura del vaso. A esta temperatura la llamaremos temperatura del agua caliente (Tc). A continuación, se vierte rápidamente el agua del vaso al calorímetro, se tapa y se agita constantemente, manteniendo el termómetro sumergido. Se anotan los valores de la temperatura en pequeños intervalos de tiempo y se elige el valor de la temperatura máxima, que se denominará temperatura de equilibrio (Teq). Puesto que el calor cedido por el agua caliente es absorbido por el agua fría y el calorímetro, haciendo un balance energético, se tiene que: –Hdesp = Habs – Hdesp = – m (agua caliente)·ce·(Teq – Tc) Habs = m (agua fría)·ce·(Teq – Ti) + K·(Teq – Ti) siendo: m: masa de agua fría = masa de agua caliente = 100 g ce: calor específico del agua = 1 cal g-1 grado-1 De este modo, se puede determinar la constante del calorímetro (K) expresada en cal/grado.  Realizar el calibrado tres veces y calcular la media aritmética del valor de K para utilizarlo en las experiencias posteriores.

3.2. Cálculo de �H1 Para determinar el calor de disolución del NaOH (s), se introducen 200 ml de agua en el calorímetro. Se agita cuidadosamente, se introduce el termómetro y se mide la temperatura hasta que ésta alcance un valor constante (Ti). A continuación, se introducen 2 g de hidróxido sódico en el calorímetro, se cierra rápidamente, se agita con una varilla de vidrio hasta completa disolución y se mide la temperatura alcanzada cuando ésta se estabilice (Tf). Con los valores de las temperaturas (Ti y Tf) y aplicando la expresión [I] se determina el calor liberado en la disolución del hidróxido sódico.

172

199

Química para Ingeniería. Problemas y prácticas



3.3. Cálculo de �H2 En esta experiencia se pretende determinar el calor de neutralización del NaOH (aq) con HCl (aq). Para ello, se vierten 100 ml de HCl 1M en el calorímetro, se introduce el termómetro y agitando cuidadosamente se mide la temperatura hasta que ésta adquiera un valor constante (Ti). A continuación, se añaden 100 ml de NaOH 0,5 M y se agita hasta alcanzar una temperatura estable (Tf). Con los datos Ti y Tf y aplicando la expresión [I] se determina el calor de neutralización del hidróxido sódico.

3.4. Cálculo de �H3 Se pretende medir el calor desprendido en el proceso de disolución del hidróxido sódico y neutralización con ácido clorhídrico. Para ello, se introducen 200 ml de HCl 1M en el calorímetro y se agita hasta alcanzar una temperatura constante (Ti). A continuación se introducen 2 g de NaOH en el calorímetro, se agita con una varilla de vidrio hasta completa disolución y se mide la temperatura alcanzada cuando ésta se estabilice Tf. Con los datos Ti y Tf y aplicando la expresión [I] se determina el calor de disolución del hidróxido sódico y neutralización con ácido clorhídrico.   - Todos los volúmenes deben ser medidos con probeta. - El NaOH (s) es una sustancia higroscópica y corrosiva. - Después de cada experiencia el material se debe lavar y secar.

4. Resultados y cuestiones 4.1. Calcular la entalpía molar de disolución del NaOH (�H1). 4.2. Calcular la entalpía molar de neutralización del NaOH (�H2). 4.3. Calcular la entalpía molar de disolución del NaOH y su posterior neutralización con HCl (�H3). 4.4. ¿Los procesos observados son exotérmicos o endotérmicos? Razonar la respuesta.

200

173



Práctica 1. Termodinámica Química

4.5. ¿La constante del calorímetro puede tener un valor negativo? 4.6. Comprobar que se cumple la ley de Hess en los procesos anteriores.

174

201



Práctica 2. Cinética Química

175



1. Introducción Al estudiar una reacción química, se pueden considerar dos aspectos: - El estático, que se refiere a la dirección y la extensión en que se da un proceso. - El dinámico, que trata del estudio de la velocidad y el mecanismo de una reacción. La Cinética es la parte de la Química que estudia el aspecto dinámico de un proceso. Se denomina velocidad de reacción a la variación de concentración de una de las especies químicas que interviene en una reacción con el tiempo. Se puede expresar como desaparición de reactivos o como formación de productos. Hay varios factores que modifican la velocidad de una reacción, éstos son: - Concentración de los reactivos: a mayor concentración mayor velocidad. - Superficie de contacto: a mayor superficie de contacto mayor velocidad. - Temperatura: a mayor temperatura mayor velocidad. - Catalizador: una pequeña cantidad del mismo modifica la velocidad.

2. Objetivos 2.1. Determinar experimentalmente la velocidad de una reacción. 2.2. Observar cómo influyen los factores anteriormente citados sobre la velocidad de una reacción.

3. Parte experimental 3.1. Medida de la velocidad media de una reacción - Pesar en una balanza una muestra de unos 0,05 g de magnesio (Mg), anotando el peso exacto de la muestra. - Añadir los 0,05 g de Mg a un tubo de ensayo que contenga 4 ml de HCl 3 M, (previamente medidos con una pipeta), a la vez que se pone en marcha un cronómetro. - Medir el tiempo que tarda en desaparecer la muestra de magnesio.

176

205

Química para Ingeniería. Problemas y prácticas



La velocidad media del proceso de disolución del Mg en el HCl se puede expresar como: v=–

  = – �Mg (mol·L

d Mg dt

�t

-1

·s-1)

[I]

3.2. Factores que afectan a la velocidad de una reacción  En dos tubos de ensayo perfectamente limpios y secos, introducir en cada uno de ellos 4 ml de HCl 0,5 M y 3 M, respectivamente. Agregar 0,05 g de magnesio a cada uno de los tubos simultáneamente. Observar en cuál de los dos tubos de ensayo el desprendimiento de hidrógeno es más rápido.    Tomar dos tubos de ensayo perfectamente limpios y secos, pesar en uno de ellos un clavo de hierro y en el otro el mismo peso del mismo metal en polvo. Agregar a cada tubo 4 ml del HCl 3 M. Observar en cuál de los dos tubos de ensayo el desprendimiento de hidrógeno es más rápido.    Preparar en un vaso de precipitados de 100 ml una mezcla formada por 10 ml de oxalato sódico (Na2C2O4) 0,1 M; 4 ml de permanganato potásico (KMnO4) 0,05 M y unas gotas de ácido sulfúrico (H2SO4). Repartir la mezcla en tres tubos de ensayo. Calentar uno de ellos. Dejar otro tubo durante un tiempo (1 h) a temperatura ambiente con el fin de poder comparar el efecto de la temperatura. Observar el aspecto de la disolución que se calienta y compararlo con respecto a la que está a temperatura ambiente.    Agregar una punta de espátula de sulfato de manganeso (II), MnSO4, al tercer tubo de la experiencia anterior. Observar el aspecto de la disolución catalizada y compararlo con respecto a la que está a temperatura ambiente. 206

177



Práctica 2. Cinética Química

 - Los vapores de HCl son muy corrosivos, por lo que las experiencias en las que se emplea HCl 3 M se deben REALIZAR EN CAMPANA EXTRACTORA.

4. Resultados y cuestiones 4.1. Calcular la velocidad media del proceso de disolución del Mg en HCl según la expresión [I]. 4.2. ¿Cómo se explica la diferencia de velocidad en los dos tubos de ensayo que contienen la misma cantidad de magnesio y distinta concentración (3 y 0,5 M) de HCl? 4.3. ¿Cómo se explica la diferencia de comportamiento en los dos tubos de ensayo que contienen el clavo de Fe y el polvo de Fe en presencia de HCl 3 M? 4.4. ¿En cuál de los dos tubos de ensayo que contienen oxalato sódico, permanganato potásico y ácido sulfúrico la velocidad de la reacción es mayor, en el que está a temperatura ambiente o en el que se calienta? ¿Por qué? 4.5. ¿En cuál de los dos tubos de ensayo que contienen oxalato sódico, permanganato potásico y ácido sulfúrico la velocidad de la reacción es mayor, en el que está a temperatura ambiente o en el que se añade el catalizador? ¿Por qué?

178

207



Práctica 3. Aplicaciones Industriales de las Reacciones de Oxidación y Reducción

179



1. Introducción Una reacción de oxidación-reducción (redox) es aquélla en la que ocurre una transferencia electrónica. Una especie cede electrones (se oxida) y otra especie los acepta (se reduce). Las celdas galvánicas o pilas son los dispositivos en los que ocurre una reacción redox espontánea. Están constituidas por dos compartimentos, conectados entre sí, llamados semiceldas. La semicelda en la que tiene lugar la semirreacción de reducción se llama cátodo y la otra, ánodo. semirreacción de oxidación

A � An+ + n e-

semirreacción de reducción

-

B+ne �B

n-

ÁNODO CÁTODO

La reacción redox global que se produce en la pila es la suma de las dos semirreacciones: A + B � An+ + BnLas celdas galvánicas o pilas sirven para almacenar energía en forma de oxidantes y reductores, que luego puede ser liberada como corriente eléctrica, impulsada por una diferencia de potencial. Entre las celdas galvánicas, la pila de hidrógeno-oxígeno es un ejemplo de celda de combustible, que ha adquirido un gran interés en los últimos años ya que permite obtener energía de forma limpia y sin generar subproductos contaminantes. En la celda de combustible de hidrógeno-oxígeno los agentes oxidantes y reductores, H2 y O2, se suministran de forma continua a los electrodos. Las celdas electrolíticas son los dispositivos en los que ocurre una reacción redox no espontánea gracias a la energía suministrada por una batería o fuente de alimentación. En los últimos años, la electrólisis del agua ha suscitado un gran interés debido a que permite la obtención del hidrógeno y el oxígeno necesarios para alimentar las celdas de combustible hidrógeno-oxígeno. Además, el proceso se convierte en sostenible cuando la energía necesaria para llevar a cabo la electrólisis del agua proviene, por ejemplo, del exceso de energía que producen las centrales eléctricas o nucleares por la noche, cuando hay menor consumo de electricidad.

2. Objetivos 2.1. Reconocer y ajustar las semirreacciones de oxidación y de reducción que tienen lugar en un proceso redox. 2.2. Medir el potencial de una pila y comparar el valor obtenido con el calculado teóricamente. 180

211

Química para Ingeniería. Problemas y prácticas



2.3. Comprobar la espontaneidad de una reacción redox y relacionarla con el valor del potencial del proceso. 2.4. Comprobar la aplicación industrial de las reacciones de oxidaciónreducción mediante el empleo de una pila de combustible de hidrógenooxígeno.

3. Parte experimental 3.1. Estudio cualitativo de la espontaneidad de una reacción redox Introducir un clavo de hierro en un tubo de ensayo que contenga 2 ml de CuSO4 diluído. Observar y anotar los cambios que tienen lugar. Con los datos de los potenciales de reducción estándar y de los colores de las especies que intervienen en el proceso, identificar los productos obtenidos y escribir las semirreacciones de oxidación y de reducción así como la reacción global. Eº (Cu2+ / Cu) = 0,34 V; Eº (Fe2+ / Fe) = - 0,44 V Cu (s)

azul

2+

verde claro

Cu Fe

rojizo

2+

3.2. Montaje de la pila Daniell y medida de la diferencia de potencial Montar la pila según el siguiente esquema:

212

Zn

Cu

[Zn 2+ ] = 1M

[Cu 2+ ] = 1M

181

Práctica 3. Aplicaciones Industriales de las Reacciones de Oxidación y Reducción 

Al conectar las láminas de Cu y de Zn con un hilo conductor tiene lugar la reacción redox espontánea. Conectar un voltímetro y medir la diferencia de potencial entre las semiceldas. Con los datos de los potenciales de reducción estándar de las especies que intervienen en el proceso, escribir y ajustar las semirreacciones de oxidación y reducción, así como la reacción global. Eº (Cu2+ / Cu) = 0,34 V

Eº (Zn2+ / Zn) = - 0,76 V

Indicar qué semicelda es el cátodo y cuál es el ánodo, así como la polaridad de los electrodos. Indicar la dirección en la que se mueven los electrones. Explicar para qué sirve el tabique poroso que hay entre las dos disoluciones.

3.3. Estudio de la espontaneidad de algunas reacciones  Realizar las reacciones que se indican a continuación en tubo de ensayo, en el medio indicado, tomando 2 ml de cada reactivo. Observar y anotar todos los cambios ocurridos:              : Introducir en el tubo la disolución de Na2SO3, 4 gotas de una disolución acuosa de ácido sulfúrico y, a continuación, la disolución de KMnO4.                    Introducir en el tubo la disolución de H2O2, 4 gotas de una disolución acuosa de ácido sulfúrico y, a continuación, la disolución de K2Cr2O7.  Identificar los productos que se forman por el cambio de color, formación de gases o reacciones específicas, tomando como referencia el color que presentan las siguientes especies: Mn2+

incoloro

MnO4-

violeta

SO42-

incoloro

SO32-

incoloro

Cr2O72- naranja Cr3+

182

verde o violeta, según la concentración

213

Química para Ingeniería. Problemas y prácticas



 Escribir la reacción de acuerdo con lo observado en el apartado anterior. Ajustarla por el método del ión–electrón. Los pasos a seguir para ajustar una reacción redox por el método ión–electrón son los siguientes: a) Identificar la especie que se oxida y la que se reduce. b) Escribir las semirreacciones de oxidación y reducción de forma separada. c) En cada semirreacción ajustar en primer lugar los átomos distintos a oxígeno e hidrógeno. Si la reacción se produce en medio ácido, los átomos de oxígeno se ajustan añadiendo H2O donde convenga, después se ajustarán los átomos de hidrógeno añadiendo H+. Por último, se ajustará la carga con electrones (e-). d) Combinar las semirreacciones para obtener la reacción global.  Comprobar teóricamente, con los potenciales de reducción estándar, la espontaneidad de la reacción. Eº (MnO4- / Mn2+) = 1,49 V; Eº (SO42- / SO32-)= - 0,92 V; Eº (Cr2O72- / Cr3+) = 1,33 V; Eº (O2 / H2O2) = 0.68 V.

3.4. Pila de combustible de hidrógeno-oxígeno En el laboratorio se va a realizar el montaje de una pila de combustible de hidrógeno-oxígeno acoplada con un electrolizador que emplea la técnica de membrana de intercambio de protones (Proton Exchange Membrane, PEM). Esta tecnología permite prescindir totalmente del uso de ácidos y emplear únicamente agua destilada. En el electrolizador o celda electrolítica el agua pura se descompone mediante electrólisis (proceso redox no espontáneo que requiere el aporte de energía) en hidrógeno y oxígeno al aplicar la energía necesaria con una fuente de alimentación. A continuación, se muestra el esquema de funcionamiento y las reacciones que se producen en el electrolizador donde el agente oxidante, la especie que se reduce, es el agua (los H+ se reducen a H2), y el agente reductor, la especie que se oxida, es también el agua (el agua se oxida a O2).

214

183

Práctica 3. Aplicaciones Industriales de las Reacciones de Oxidación y Reducción 

-

+ Batería

4e

-

(-) CÁTODO

4e

-

ÁNODO (+)

2 H2

4H

O2

+

2 H2O

Reacción del ánodo + 2HO4e +4H +O

Reacción del cátodo + 4 H + 4 e  2 H2



2

2

PEM = Membrana de intercambio de protones



Esquema del electrolizador Utilizando la ecuación: Q = I·t

[I]

donde Q: carga (Culombios) I: Intensidad de corriente (Amperios) t: tiempo (segundos) y teniendo en cuenta que 1 mol de electrones genera una carga eléctrica de 96500 C, se puede calcular el número de moles teóricos de H2 y O2 que se producen en el electrolizador aplicando una intensidad de corriente determinada durante un tiempo dado.

184

215

Química para Ingeniería. Problemas y prácticas



A continuación se muestra el esquema de funcionamiento y las reacciones que se producen en la celda de combustible de hidrógeno-oxígeno, donde el agente oxidante, la especie que se reduce, es el oxígeno, y el agente reductor, la especie que se oxida, es el hidrógeno. -

4e



-

4e



ÁNODO

CÁTODO

2 H2

4H

Pt

Reacción del ánodo + 2 H2 4 H + 4 e

O2

+

2 H2O

Pt

Reacción del cátodo + 4 e + 4 H + O2  2 H2O 

PEM = Membrana de intercambio de protones

Esquema de la celda de combustible de hidrógeno-oxígeno En el ánodo, el hidrógeno se oxida a protones y en el cátodo el oxígeno se reduce a agua. La reacción es espontánea, tal y como se puede deducir del valor de los potenciales redox de las especies que participan en la reacción (Eº (H+ / H2) = 0 V; Eº (O2 / H2O) = + 1.23 V) y genera electricidad, calor y agua. La energía eléctrica generada por la pila de combustible alimenta en este caso un pequeño motor. El oxígeno y el hidrógeno necesarios para alimentar la pila de combustible provienen del electrolizador. El electrolizador se puede conectar a una batería o a

216

185

Práctica 3. Aplicaciones Industriales de las Reacciones de Oxidación y Reducción 

una celda solar de modo que la instalación completa puede ser accionada por energías renovables, en este caso, energía solar.     Realizar el montaje del electrolizador como se indica en la figura: 4

1

3 5

2

1. Generador de corriente. 2. Electrolizador PEM. 3. Tanques de agua y contenedores de gases. 4. Medidor de gases. 5. Conectores entre los tanques de agua y el medidor de gases. Llenar los erlenmeyer del medidor de gases (4) a través del cilindro de plástico con agua destilada hasta que el erlenmeyer y el tubo de vidrio (en ángulo recto) estén llenos de agua y no contengan burbujas. Unir los conectores (5) (tubos de goma) entre el medidor de gases (4) y los tanques de agua (3). Llenar los 186

217

Química para Ingeniería. Problemas y prácticas



tanques de agua (3) con agua destilada hasta la línea que indica “Gas test”. Cerrar los tanques de agua y contenedores de gases (3) con un tapón de modo que el gas no pueda escapar. Conectar el electrolizador (2) al generador de corriente (1), estableciendo una intensidad de corriente entre 1,6 y 1,8 A y un voltaje entre 1,7 y 2 V, y poner en marcha el cronómetro. Anotar el tiempo cuando se han desprendido 25 y 50 ml de H2. Observar en el cilindro de plástico del medidor de gases (4) que se genera doble volumen de H2 que de O2.

t (s)

V H2 (ml)

V O2 (ml)

25

12,5

50

25

0RQWDMHGHXQDSLODGHFRPEXVWLEOHGHKLGUyJHQRR[tJHQR Realizar el montaje como se indica en la figura: 4 2 7

1

5

6

3

1. Lámpara, fuente de irradiación. 2. Placa solar, fuente de energía que alimenta el electrolizador.

218

187

Práctica 3. Aplicaciones Industriales de las Reacciones de Oxidación y Reducción 

3. Electrolizador PEM. 4. Tanques de agua y contenedores de gases. 5. Conectores entre el electrolizador y la pila de combustible que transportan el H2 y el O2. 6. Pila de combustible PEM. 7. Motor accionado por la energía producida en la pila de combustible. Llenar los tanques de agua (4) con agua destilada hasta un nivel intermedio entre las marcas de máximo y mínimo. Conectar la placa solar (2), en nuestro caso una batería, al electrolizador (3). Conectar los tubos de goma que salen de los tanques de agua (5) a la pila de combustible (6). Conectar la pila de combustible (6) al motor (7) (y/o a un voltímetro para medir el voltaje generado). Una vez realizado el montaje, enchufar la lámpara y dejar funcionar el sistema durante unos minutos. Al cabo de unos minutos, el motor se pondrá en movimiento accionado por la energía proporcionada por la pila de combustible, que a su vez ha sido alimentada por el electrolizador. En las fotos se muestra el montaje de la pila de combustible de hidrógenooxígeno antes (arriba) y después (abajo) de conectarla a una fuente de alimentación. Notar que antes de conectarla el motor está parado, mientras que al ponerla en funcionamiento el motor está en movimiento.

188

219

Química para Ingeniería. Problemas y prácticas



4. Resultados y cuestiones 4.1. a) ¿Qué cambios se observan en el transcurso de la reacción entre el clavo de hierro y la disolución de CuSO4? b) Escribir la reacción que tiene lugar. 4.2. a) Calcular el potencial de la pila Daniell y comparar el valor medido experimentalmente con el teórico. Comprobar la espontaneidad de la reacción. b) Si sustituimos la semicelda de Cu por un hilo de plata sumergido en una disolución de AgNO3 1 M, indicar cuál será en este caso el potencial de la pila. Datos: Eº (Ag+ / Ag) = 0,8 V; Eº (Zn2+ / Zn) = - 0,76 V 4.3. a) Describir los cambios observados durante la reacción de Na2SO3 con KMnO4 en medio ácido. b) Ajustar las semirreacciones y la reacción global. c) Calcular el potencial estándar de la reacción. 4.4. a) Describir los cambios observados durante la reacción de H2O2 con K2Cr2O7 en medio ácido. b) Ajustar las semirreacciones y la reacción global. c) Calcular el potencial estándar de la reacción.

220

189

Práctica 3. Aplicaciones Industriales de las Reacciones de Oxidación y Reducción 

4.5. Escribir las reacciones redox globales que se producen en el electrolizador y en la pila de combustible. Indicar cuál es la principal diferencia entre ambas. 4.6. Razonar por qué se genera doble volumen de H2 que de O2 en el electrolizador. 4.7. Utilizando la ecuación [I], calcular el volumen teórico de H2 y O2 que se debería producir en el electrolizador, aplicando una intensidad de corriente de 1,8 A durante los distintos tiempos que se han obtenido experimentalmente. Rellenar para ello la siguiente tabla, considerando que los gases se comportan como gases ideales en condiciones estándar (1 atm, 25 ºC).

t (s)

moles e-

moles H2

V H2 (ml)

moles O2

V O2 (ml)

4.8. Comparar el volumen de O2 ó H2 medido experimentalmente en la práctica con los volúmenes calculados teóricamente y determinar la eficiencia del electrolizador a los distintos tiempos, empleando para ello la siguiente expresión:

eficiencia =

 V O2 ó H2 experimental 

. 100

V O2 ó H2 teórico 

190

221



Práctica 4. Relación entre Estructura Molecular y Propiedades Físicas de los Compuestos Químicos

191



1. Introducción Las fuerzas de atracción entre las moléculas, que se denominan fuerzas intermoleculares, son importantes porque de ellas dependen las propiedades físicas de los compuestos. Estas interacciones se deben a la distribución de la densidad electrónica en las moléculas que puede ser permanente o momentánea. En base a esta distribución se conocen distintos tipos de fuerzas. En la tabla que se muestra a continuación se indican los diferentes tipos de fuerzas intermoleculares ordenadas de menor a mayor intensidad. Energía (kJ/mol)

Tipo de compuestos

Ejemplo

0,05-40

Covalentes

CH4

Dipolo-dipolo

5-25

Covalentes polares

CH3Cl

Enlace de hidrógeno

10-40

Covalentes polares con H unido a F,O o N

CH3OH

Ión-dipolo

40-600

Iónicos en agua

K+ / H2O

400-4000

Iónicos

NaCl

Tipo de fuerza

+

Dispersión (London)

Ión-ión

En general, las fuerzas intermoleculares determinan las propiedades físicas de los compuestos como son el estado de agregación, el punto de fusión y de ebullición y la solubilidad. Así, el estado sólido es característico de compuestos con fuerzas intermoleculares fuertes como los compuestos iónicos (NaCl, Na2CO3, etc.). Algunos compuestos covalentes presentan también estado de agregación sólido, a pesar de tener interacciones débiles del tipo de fuerzas de London, ya que estas fuerzas son mayores a medida que aumenta el tamaño de las moléculas del compuesto. Compuesto Hidrocarburos Halógenos

192

Estado de agregación Gas

Líquido

Sólido

1�C 40

F2, Cl2

Br2

I2

225

Química para Ingeniería. Problemas y prácticas



Cuando las interacciones entre las partículas son débiles, las moléculas pueden alejarse unas de otras y se manifiesta el estado de agregación gaseoso. A medida que las interacciones entre las partículas van aumentando, permiten que las moléculas se mantengan próximas; se presenta el estado líquido y, a mayor intensidad, se llega a manifestar el estado sólido. Otras propiedades físicas que también están relacionadas con las fuerzas intermoleculares son la temperatura de fusión (Tf) y de ebullición (Teb) de un compuesto. Para que un sólido funda o un líquido se vaporice, las fuerzas que mantienen unidas a las moléculas deben romperse. Los compuestos con fuerzas de atracción muy débiles (como las de dispersión) tienen puntos de fusión (Pf) y de ebullición (Peb) muy bajos, ya que sólo se necesita una pequeña cantidad de energía para separar las moléculas y formar así un líquido o un gas. Los compuestos que poseen enlaces de hidrógeno o interacciones dipolodipolo necesitan más energía para vencer las fuerzas atractivas entre sus moléculas. Por último, los mayores puntos de fusión y ebullición se observan en los compuestos iónicos. Sustancia

Peb (ºC)

Sustancia

Peb (ºC)

CH3CH2OH

78

SH2 (gas)

- 60

H2O (líquido)

100

CH3OCH3

- 25

CH3CH2Cl

13

2. Pruebas de solubilidad 2.1. Introducción La solubilidad se define como la capacidad que presenta un soluto para disolverse en un disolvente a una temperatura específica. Se trata de una propiedad física que depende de la estructura química de ambos compuestos, por lo tanto depende de las fuerzas intermoleculares entre el soluto y el disolvente. Cuando una sustancia se disuelve en otra, el resultado es una disolución, una mezcla homogénea. La similitud estructural suele ser un criterio acertado para predecir si un compuesto es o no soluble en un determinado disolvente. La estructura molecular condicionará el tipo de interacciones intermoleculares que se producirán entre el soluto y el disolvente y permitirá la solubilidad o no entre ambos. Las

226

193

Práctica 4. Relación entre Estructura Molecular y Propiedades Físicas de los Compuestos Químicos 

sustancias con estructuras moleculares parecidas tienen probabilidades de presentar fuerzas intermoleculares semejantes y de ser solubles entre sí. Las sustancias con estructuras diferentes tienen dificultad para formar disoluciones. El criterio de solubilidad se puede enunciar como “lo semejante disuelve a lo semejante” y está íntimamente relacionado con las atracciones entre las moléculas de soluto y de disolvente. Cuando un soluto se disuelve, las atracciones soluto-soluto son reemplazadas por las atracciones soluto-disolvente. Si las nuevas atracciones son similares a las reemplazadas, se formará la disolución; pero si el balance de energía es desfavorable no habrá disolución. A continuación, se muestran los distintos casos que se pueden encontrar entre un soluto y un disolvente según la naturaleza molecular de los compuestos. Así, un líquido polar, como el agua, suele ser el mejor disolvente para los compuestos iónicos y polares.

CH3OH

CH3OH

CH3OH

CH3OH

CH3OH

CH3OH

SOLUTO Polar

CH3OH CH3OH

CH3OH CH3OH DISOLVENTE Polar

DISOLUCIÓN

Por el contrario, los líquidos no polares, como el hexano, son buenos disolventes para los compuestos no polares.

Cn H2n CnH2n CnH2n

CnH2n

Cn H2n CnH2n

Cn H2n

CnH2n

SOLUTO Apolar

DISOLVENTE Apolar

CnH2n CnH2n

DISOLUCIÓN

En el siguiente ejemplo se muestra la imposibilidad de disolución de un soluto apolar, como el hexano, en un disolvente polar, como el agua, debido a la diferencia estructural entre ambos.

194

227

Química para Ingeniería. Problemas y prácticas

CnH2n CnH2n CnH2n



CnH2n

CnH2n

CnH2n

CnH2n

CnH2n

CnH2n

SOLUTO Apolar

CnH2n CnH2n

DISOLVENTE Polar

NO DISOLUCIÓN

Teniendo en cuenta la regla de “lo similar disuelve lo similar” podemos entender el comportamiento de los tensoactivos o surfactantes, como son el jabón y los detergentes sintéticos. Los jabones son sales de sodio o potasio de ácidos carboxílicos de cadena larga, cuya estructura se muestra en la siguiente figura: COO

Na

Parte hidrófila

Parte lipófila

El extremo en el que se encuentra el grupo carboxilato polar (cabeza) constituye la parte hidrófila que tiende a disolverse en agua, y la cadena hidrocarbonada no polar (cola) es la parte hidrófoba y tiende a disolverse en la grasa. En este sentido, el jabón es muy eficaz para eliminar la grasa porque, en presencia de agua, forma micelas que envuelven a las partículas de grasa en su interior y las mantienen en suspensión en la disolución acuosa. Este fenómeno se debe a la orientación de la zona hidrófila de la cadena del jabón hacia el exterior de la micela, como se muestra en el esquema:

O

O

O

O O

O Partícula de grasa (insoluble en agua)

O O

O O

O

O

Micela de detergente (soluble en agua)

228

O

O

O

O O

O

O O

O O

O

O

Grasa en suspensión

195

Práctica 4. Relación entre Estructura Molecular y Propiedades Físicas de los Compuestos Químicos 

El papel limpiador de un detergente se debe no sólo a la capacidad que tiene de disolver la grasa en el agua, sino también a la propiedad que presenta de disminuir la tensión superficial del agua favoreciendo el contacto entre el agua y el aceite (o grasa). Las sustancias que presentan esta segunda propiedad se denominan tensoactivos y poseen aplicaciones técnicas importantes relacionadas con su poder detergente, emulgente y espumógeno.

2.2. Objetivos 2.2.1. Comprobar la solubilidad de distintos compuestos en agua, y justificar su comportamiento en base a las estructuras moleculares y las fuerzas intermoleculares. 2.2.2. Demostrar el papel que ejerce un jabón en una mezcla de aceite-agua y explicar su funcionamiento.

2.3. Parte experimental En una gradilla disponer 3 tubos de ensayo y añadir, aproximadamente, 2 ml de disolución acuosa de CuSO4 a cada uno de ellos. A continuación, añadir 2 ml de etanol en el primero, 2 ml de acetona en el segundo y 2 ml de hexano en el tercero. Observar la solubilidad de cada compuesto orgánico en la disolución acuosa (coloreada) y anotar los cambios físicos que experimentan las mezclas. Por otra parte, en un vaso de precipitados de 50 ml añadir, aproximadamente, 20 ml de agua destilada y 1 g de cloruro sódico. Observar la solubilidad del compuesto sólido en agua. A continuación, preparar otros dos tubos de ensayo con, aproximadamente, 2 ml de agua destilada y 10 gotas de aceite. Seguidamente, añadir a uno de ellos 10 gotas de jabón líquido. Observar la variación de la solubilidad del aceite en agua en cada tubo.

196

229

Química para Ingeniería. Problemas y prácticas



2.4. Resultados y cuestiones 2.4.1. Describir el aspecto físico del contenido de todos los tubos. 2.4.2. Justificar las diferencias de comportamiento en cuanto a la solubilidad de los diversos compuestos en disolución acuosa en base a sus estructuras e interacciones intermoleculares. 2.4.3. Explicar el papel que ejerce el jabón en la solubilidad del aceite en agua.

230

197

Práctica 4. Relación entre Estructura Molecular y Propiedades Físicas de los Compuestos Químicos 

3. Prueba comparativa de acidez 3.1. Introducción La acidez es una propiedad de ciertos compuestos orgánicos que se encuentra muy relacionada con su estructura molecular y, por lo tanto, con su solubilidad. Así, la mayor parte de las sustancias orgánicas no son solubles en agua pero, si mediante una reacción química se produce un cambio en su estructura, pueden convertirse en especies solubles. Por ejemplo, cuando una sustancia ácida poco soluble en agua reacciona con una base, cediéndole un protón, se convierte en una especie iónica soluble en agua. La capacidad de un ácido para ceder un protón se mide a través de su constante de acidez (Ka). Cuanto menor es el valor de la constante de acidez, más débil es el ácido. La mayoría de los ácidos y de las bases que existen en la naturaleza son débiles (Ka